Reply
 
Thread Tools Display Modes
  #41   Report Post  
Posted to rec.audio.high-end
[email protected] dpierce.cartchunk.org@gmail.com is offline
external usenet poster
 
Posts: 334
Default Ultrabit Platium Disk Treatment

On Jul 25, 9:52 pm, Codifus wrote:
wrote:
You want proof that it works? Try the following experiment:


Find two pairs of the same speakers that both suffer, to
equal degrees, the problem you claim. Allow someone
to take ONE pair: you don't know which, and apply or NOT
apply the Densen CD treatment to it, completely out of
your site.


Afterwards, your job is very simple: all you nkow is that
one of the two pairs MAY have had the treatment applied.
BY LISTENING ALONE, tell us which one it is.


Do that reliably in a number of cases, and then, maybe,
it gets interesting. Otherwise, yawn.

This is an even better setup. I am totally game.


Fine, and I would further expand the experimental
design as follows: you have two otherwise identical pair
of loudspeakers. One of them may or may not have
been exposed to a CRT-based TV. You don't know
ahead of time which is which. BY SOUND ALONE,
tell us which pair it is.

But if it works for you, why do you even care what other
people think, and thus why even bother engaging in the
discussion, unles you DO want to face a skeptical
audience?

Why do I care? I guess I like to share. I subscribe to
rec.audio,high-end to discuss hifi with others that
tend to share the same enthusiasm for audio.
Why do you post on rec.audio,high-end?


I was one of the half dozen or so original founding
members of the group back in 1989 when it was
started by Tom Krueger as a mail list. I was invited
to participate precisely because of my background
in loudspeakers, acoustics, electronics, hi fi and
physics.

  #42   Report Post  
Posted to rec.audio.high-end
[email protected] dpierce.cartchunk.org@gmail.com is offline
external usenet poster
 
Posts: 334
Default Ultrabit Platium Disk Treatment

On Jul 25, 10:56 pm, Codifus wrote:
Yes, I completely understand.


In fact, I believe the evidence you've presented shows
that you don't, with all respect.

When you move speakers past the TV, this
red and blue appears across the screen.
The effect of the speaker on the TV is much
more noticeable than any effect that may be
on the speaker. But there is an effect.


Again, you state this as if it was accepted fact.
It is not.

The CD is playing a tone. That tone manifests
itself as an electrical signal that goes through
your entire audio system. That electrical signal in
turn generates a magnetic field. The properties
of that generated magnetic field are such that it
neutralizes any static magnetism that may have
developed in the system.


Sorry, this is so much technobabble.

Why and how is this signal in any way different
than EVERY OTHER SIGNAL that passes through
an audio system that integrates to 0?

Please explain how the VAST majority of materials
in an audio system "develop a static magnetism"
when they, in fact, do not posses a net polar magnetic
moment, materials such as copper, aluminum, plastic,
glass and such. I run a current by them of hundred,
thousands of amps, and they do NOT "develop a static
magnetism," how is it possible for current of millionths
of an amp to do it.

And how is it possible for those materials that COULD
"develop a static magnetism" (irons and steels) to
then have that reversed by currents of millionths of
an amp, far too small to overcome the coercivity in such
materials?

You know, just as Arny Kruger mentioned deguassing
old real to reel tapes with a degausser and slowly
moving away, this demagic CD is doing the exact same
thing.


No, it's not, and this is further evidence that you do
NOT understand it. It;s substantially different in at
least one important factor: the field generated by a
degausser if MILLIONS of time stronger than the field
generated by the currents inside your audio system.
It HAS to be to overcome the coercivity of the magnetic
material in the tape. If the impressed field DOES NOT
exceed a critical threshold by a wide margin, no change
in the magnetization of the material occurs. And a few
microamps of signal passing through an audio system,
even a few amps passing through a voice coil is FAR too
small to work.

Think of the the tone as analogous to the
degausser's magnetic field. Now, as you
slowly move away from the tape
heads the Demagic tone slowly fades away.

That's all it is.


And you fell for such a simple, intuitive explanation
hook, line and sinker.

Too bad the explanation is simply wrong.

  #43   Report Post  
Posted to rec.audio.high-end
[email protected] dpierce.cartchunk.org@gmail.com is offline
external usenet poster
 
Posts: 334
Default Ultrabit Platium Disk Treatment

On Jul 25, 7:10 pm, Codifus wrote:
OK. Basic law of physics: every electrical field has an associated
magnetic field and vice versa. One does not exist without the other.
This is the exact principle behind transformers, speakers, you name it.
Electromagnetism.


And precisely how is that relevant to your claim?
YOU'RE the one invoking electromagnetism, YOU
must have some good reason for doing so.

If you push an electric current through a speaker,
it moves. If you move the speaker cone with your
hand, guess what? An electrical current will
be generated that you can measure at the speaker
terminals.


So? How is that relevant to your claim?

I'm more than willing to prove it to anyone. This test is so easy its
ridiculous.


Than you should find it easy, using the protocol
I suggested, to prove it. Go and do so.

Any skeptical taker who has the respect of Dick Pierce, Arny Kruger,
Sonnova, or Steve Sullivan. I chose you 4 because you have always been
so adamant about your beliefs based on facts.


Please do not lump me in with others. Your
understanding of science is demonstrably poor,
and thus your characterizations and conclusions drawn
from that poor model are unreliable.

I'm just trying to open some minds.


Problem is, open your mind enough and
anything can fall in. And out.

Now, that taker should have a pair of non-
magnetically shielded speakers. I suggest
these type of speakers because they are
easily set out of whack by moving them
around near a strong magnetic field like
a CRT based TV.


You state this as if it was accepted, axiomatic
fact.

Precisely on what basis do you make this
extraordinary claim? CHanges in the magnetic
properties of loudspeakers are trivially easy to
measure, yet in all the laboratories and referreed
scientific journals that deal with the magnetic
properties of materials, no where in the the entire
professional world of audio engineering is there
the SLIGHTEST HINT that unshielded speakers
are "easilt set out of whack near a stron magnetic
field like a CRT based TV."

Not one.

Why is that?

How "strong" do you think the magnetic field near a
TV is? Do you know? I do, because it is my job to
measure such fields.

And2 feet away from a CRT based TV, the external
field of that TV is INDISTINGUISHABLE FROM THE
BACKGROUND DUE TO THE EARTH'S MAGNETIC
FIELD.

That's approximately 0.5 gauss at mid lattitudes.

So, if the field in the vicinity of a CRT-based TV is
onthe order of the same strength of the earth's
magnetic field, than, if CRT's affect speakers, the
earth's magnetic field sghould affect them thus
there should be NO difference between speakers
due to their proximity to CRT TV's

Yet you claim there are such differences.

Typical unshielded speakers can withstand
external static and alternating fields of HUNDREDS
of times higher than this with NO change in the
magnetic properties, and such fields are HUNDREDS
of times higher than devices such as motors or TVs
radiate.

YOU have made a specific assertion he

I suggest these type of speakers because
they are easily set out of whack by moving them
around near a strong magnetic field like a CRT
based TV.

Thus, given the tremendous amount of knowledge
and experience that proceeds us all about such things,
in and of itself, constitutes and extraorindary claim.
Regardless of what sonic effects you claim result from
this, this statement alone is an exytraordinary claim that
can either be shown by PHYSICAL evidence to be
true or false.

I have, in fact, gathered data on THOUSANDS of
speakers, including magnetic data. And, along with
others we have at hand data on MILLIONS of speakers
that refute this one claim.

You have presented NOT A SINGLE SHRED of data
so support the PREMISE of your claim, that:

they are easily set out of whack by moving them
around near a strong magnetic field like a CRT
based TV.

Not one iota.

Don't tell me anout physics, my friend. Don't
tell me how it makes sense. Don't start spewing
electromagnetic "theory" or, with all due respect, your
very poor understanding of it.

Show me ANY objective data that supports your premise
that.

they are easily set out of whack by moving them
around near a strong magnetic field like a CRT
based TV.

And, by the way, your "offer" in the form of a wager
is not just a little insulting.

You want my time to prove or refute your claim?
My rates are $150/hr, the same that I charge anyone
else for short-term consulting.
  #44   Report Post  
Posted to rec.audio.high-end
Sonnova Sonnova is offline
external usenet poster
 
Posts: 1,337
Default Ultrabit Platium Disk Treatment

On Fri, 25 Jul 2008 19:56:53 -0700, Codifus wrote
(in article ):

Sonnova wrote:
On Fri, 25 Jul 2008 16:10:10 -0700, Codifus wrote
(in article ):

Sonnova wrote:
On Fri, 25 Jul 2008 05:19:54 -0700, Codifus wrote
(in article ):

Steven Sullivan wrote:
Codifus wrote:
I also have Densen's DeMagic CD. Again, many people say its effect is
bogus. I've tried it and definitely noticed a difference. The effect
was
most apparent only the 1st time I used it, though. After that, I've
used
it every couple of months and haven't noticed a change. Perhaps I
haven't given my electronics enough time to get magnetically out of
alignment. It's preventive maintenance, then.
It's more likely sighted bias.

How does sighted bias explain that I noticed the effect the 1st time but
not any other time? All those times I played the Densen CD my sight was
good and the Densen CD looked the same

That's a rather quick and hasty dismissal, don't ya think?

CD

Please explain the theory on how a commercially stamped CD (which has no
magnetic properties) can be affected by demagnetizing? Most likely, you
hear
a difference because you WANT TO, having spent the money on the device.
subsequent listenings do not elicit the same response as the first
because
you have already heard the difference that you wanted to hear the first
time
and that your expectations are that once treated, the disc shouldn't
change.
Therefore it doesn't.
OK. Basic law of physics: every electrical field has an associated
magnetic field and vice versa. One does not exist without the other.
This is the exact principle behind transformers, speakers, you name it.
Electromagnetism.


But a CD doesn't have an "electrical field" associated with it. It's just a
piece of plastic with some aluminum sputtered onto it.

If you push an electric current through a speaker, it moves. If you move
the speaker cone with your hand, guess what? An electrical current will
be generated that you can measure at the speaker terminals.

I'm more than willing to prove it to anyone. This test is so easy its
ridiculous.

I will not use my system or CD. I can already see that if anything of
mine were used in the test, the conclusion would be that I rigged it
somehow.


nobody is arguing what you say about speakers, above. It's fundamental
electrical theory. Where I am skeptical, is what you say about about being
able to affect the magnetism of a permanent magnet speaker by its proximity
to household magnetic fields. My own experience and what little I know
about
permanent magnets says that this is wrong. Secondly, I do not believe that
degaussing a CD will have any more effect on it than degaussing a vinyl
record would or could affect it * and for the same reason. There is no
magnetically permeable materials in either. You'd have a better chance
degaussing your brain, at least blood has iron in it!
Here's the deal:

Any skeptical taker who has the respect of Dick Pierce, Arny Kruger,
Sonnova, or Steve Sullivan. I chose you 4 because you have always been
so adamant about your beliefs based on facts. Nothing wrong with that at
all. It is a completely logical way to think. I'm just trying to open
some minds. Now, that taker should have a pair of non-magnetically
shielded speakers. I suggest these type of speakers because they are
easily set out of whack by moving them around near a strong magnetic
field like a CRT based TV.


first of all, as I said before, their are no strong magnetic fields
associated with a CRT TV. Yes there is a magnetic coil called a yoke around
the neck of the CRT, but it's field is concentrated to the area inside the
neck and is used to bend the electron beam shooting down the neck at the
video raster rate. More than a few inches from that coil yields no
discernible field. I suspect that you are confused because magnetically
shielded speakers are often associated with video. This is not because a
magnetic field from the CRT might interfere with the speaker, but rather
because the speaker's magnetic field can affect the CRT.

Yes, I completely understand. When you move speakers past the TV, this
red and blue appears across the screen. The effect of the speaker on the
TV is much more noticeable than any effect that may be on the speaker.
But there is an effect.


I'd have to see some real research on that. If a 1/2-inch tape degausser
won't demagnetize a speaker over a period of years, I daresay nothing you
could put near it would either.

Often when one puts
large speakers with heavy magnets next to a TV, one gets a rainbow of
colors
not associated with the picture near the edges of the screen closest to the
speakers. By shielding the speakers' magnetic field, the chances that it
can
affect the CRT's convergence and purity is reduced.



Speakers that fail the mono test can also be used, but things get murky
and subjective. Who says that the speakers are not phase coherent, you
me? Who to believe?


Speaker phase coherence (or time alignment), ala E.H. Long is something
else.

I think you're missing the point. The CD is not being degaussed. The CD
is playing a tone. That tone manifests itself as an electrical signal
that goes through your entire audio system. That electrical signal in
turn generates a magnetic field. The properties of that generated
magnetic field are such that it neutralizes any static magnetism that
may have developed in the system.


Ah, I thought you were talking about one of thease CD degaussers that are
being sold. Silly me. OTOH, there is little or nothing about a tone played
through a system that would have any affect on that system either. It is
possible to degauss a phono cartridge using a degausser because the close
proximity of the permeable parts of the moving assembly are constantly in a
strong (if small) magnetic field.

You know, just as Arny Kruger mentioned deguassing old real to reel
tapes with a degausser and slowly moving away, this demagic CD is doing
the exact same thing. Think of the the tone as analogous to the
degausser's magnetic field. Now, as you slowly move away from the tape
heads the Demagic tone slowly fades away.

That's all it is.

CD


I'd like to see some double-blind or ABX tests on this.

  #45   Report Post  
Posted to rec.audio.high-end
[email protected] dpierce.cartchunk.org@gmail.com is offline
external usenet poster
 
Posts: 334
Default Ultrabit Platium Disk Treatment

On Jul 25, 6:01 pm, Codifus wrote:
This is where I stop stroking your ego


Don't be insulting, please, you are NOT up to the task.

Do you in fact think you know everything?


Never made the claim, did i?

Also, could you be wrong in some things
you've studied?


Indeed, as could you.

But I have studied the operation of lopudspeakers
FAR MORE than you have. I have, in fact, studied
this particualr aspect of loudpsekaers FAR more
than you have.

Unless you have some earthshaking revelation
to the contrary, that might suggest your chances
of being wrong on this topic are substantially
greater than mine, no?

Just because you don't hear something


Never even climed one way or the other: YOU'RE
claiming the presence of physical effects that would
be trivailly easy to detect by objective means, effects
that have NEVER ONCE been detected in the
millions of measurements over many decades
and you provide not a single shred of evidence.
You make claims as if they were established
fact without a shred of evidence.

or know of no law of physics to support a
claim, does not make it untrue.


It's not up to me to support your extraordinary claim:
that's YOUR jobn, and one that you're not doing a
very good job of.

It just means that it hasn't been verified.


And that's YOUR job, sir, not mine.

Hop to it.

The world was once flat. Now it's round.


Boy, you picked the WRONG model.

The vast majority of the people in any know in
the world have known the world was not flat for
over 2000 years. Archimedes, in fact, calculated
the diameter of the earth to within a feqw percent
2 millenia ago. There was NO serious quesrtioning
of this information by anyone over that period of
time.

The ONLY ones who did NOT believe the world was
round where the uneducated and the religiously
superstitious, quite analogous to those who are not
well educated in electromagnetic theory who believe
in CD tweaks and the like, e.g., many high-end
audiophiles

Sorry, you raised the comparison, I only finished it.

The titanic was unsinkable.


Not according to the designers. They were able to
describe ahead of time several sceanrios where
the ship's integrity could be seriously compromised,
including ones not unnlike that that occurred. The
only ones who truly "believed" the ship was unsinkable
were those who didin't understand ship design and
physics, not unlike audiphiles who do not understand
electromagnetic theory that make wild claims about
how CDs can restore speakers.

Sorry, you raised the comparison, I only finished it.

Light travels in a straight line.


Indeed it does: Einstein showed this conclusively
in the general theory of relativity: all objects follow
constant geodesics and APPEAR to curve due
to the curvature of space time in the presence of
massive objects. The only people who BELIEVE
otherwise are those who do not understand relativity.
not unlike audiphiles who have a poor understanding
of eletctromagnetic theory and believe that "special"
signals can eliminate "static magnetism" inside audio
components.

As man has developed more sensitive and capable
measuring instruments, things that were once un-
true became true.


And thos einstruments have existed, in the case of
what you're talking about, for well over half a century.

Yeah, still interesting.

There's another thread discussing whether we need science in audio
reviews and and now I see very clearly that the answer is most
definitely yes. There's 2 groups. The scientists, such as yourself, Arny
Kruger, and Steve Sullivan, who seem to go by the mantra that if they
can't hear it, and it hasn't been proven, then it doesn't exist. Period.


Learn to read, sir, because that is NOT was I've said.

The other group would be the reviewers such as tnt-
audio, stereophile, etc. These 2 groups have a yin/yang
relationship and it keeps both sides on their toes.


No, they don't.

Please name a SINGLE one of these "magic" and
"marvelous" audiophile innovations that has made ANY
change WHATSOEVER in mainstream science, that
has lead to ANY change in ANY paradigm of physics,
that has not died a death of ridiculed obscurity it so
richly deserved.

This CD demagnetizer nonsense has been around AT
LEAST for a decade or more, and has gotten NO traction
ANYWHERE other than in a narrow niche in the high-end
market.

Why do you suppose that is?


  #46   Report Post  
Posted to rec.audio.high-end
Arny Krueger Arny Krueger is offline
external usenet poster
 
Posts: 17,262
Default Ultrabit Platium Disk Treatment

"Codifus" wrote in message


I'm just trying to open some minds.


I long ago learned that having an open mind is not facilitated by having
holes in the head. ;-)

Now, that taker should have a pair of
non-magnetically shielded speakers. I suggest these type
of speakers because they are easily set out of whack by
moving them around near a strong magnetic field like a
CRT based TV.


You can't hurt the speaker with the TV, but you may hurt the TV with the
speaker.

Dick Pierce just showed that with his comments on my comment about
magnetizing speakers via their pole pieces. If you can't magnetize or
demagnetize a speaker via a special fixture applied to its pole pieces, you
sure as the dickens aren't going to hurt it by putting it near a TV!

Usually the TV has the resources to fix itself in the form of a built-in
degaussing coil, often automatically engaged during power up, or via a
service function on some menu some place.

  #47   Report Post  
Posted to rec.audio.high-end
Arny Krueger Arny Krueger is offline
external usenet poster
 
Posts: 17,262
Default Ultrabit Platium Disk Treatment

"Codifus" wrote in message


You know, just as Arny Kruger mentioned deguassing old
real to reel tapes with a degausser and slowly moving
away,


I didn't say that, Sonnova did.

this demagic CD is doing the exact same thing.


It can't because it lacks the ability to create an equally intense magnetic
field.

  #48   Report Post  
Posted to rec.audio.high-end
jwvm jwvm is offline
external usenet poster
 
Posts: 336
Default Ultrabit Platium Disk Treatment

On Jul 25, 5:58*pm, Sonnova wrote:
Actually, they have absolutely nothing to do with each other. There is
nothing magnetic in a stamped, mass produced CD that would have the slightest
affect on it. I was merely showing that one can magnetize and demagnetize
ferric objects using an AC signal , CDs aren't ferrous (except maybe
recordable ones which, if I understand correctly, do use a magnetic field to
effect the burning).


Its hard to follow what your problem is with Arnie's remarks. Your
claim that it is possible to magnetize steel objects with an AC field
is true but irrelevant to the original claim regarding speakers given
their physical properties. The amount and polarity of the amount of
magnetism induced is quite random with an AC field. Indeed, it is
well-known that tape heads can be magnetized if one uses a
demagnetizer incorrectly.

Also, please note that CDRs and CDRWs do not employ magnetism when
recording. You are confusing them with magneto-optical media like
minidiscs:

http://en.wikipedia.org/wiki/Magneto-optical_drive

That would be more similar to anything you might achieve by playing a CD
through a stereo, except that the Weller iron operates with far higher
currents than a speaker voice coil.


They have nothing in common.


That is not true. The voice coil and the coil attached to a soldering
gun both generate an AC field. But as you seem to have indicated, the
AC field generated by a voice coil under normal operation will not
change the characteristics of a speaker in any significant manner.
  #49   Report Post  
Posted to rec.audio.high-end
Sonnova Sonnova is offline
external usenet poster
 
Posts: 1,337
Default Ultrabit Platium Disk Treatment

On Sat, 26 Jul 2008 08:43:43 -0700, wrote
(in article ):

On Jul 25, 6:01 pm, Codifus wrote:
This is where I stop stroking your ego


Don't be insulting, please, you are NOT up to the task.

Do you in fact think you know everything?


Never made the claim, did i?

Also, could you be wrong in some things
you've studied?


Indeed, as could you.

But I have studied the operation of lopudspeakers
FAR MORE than you have. I have, in fact, studied
this particualr aspect of loudpsekaers FAR more
than you have.

Unless you have some earthshaking revelation
to the contrary, that might suggest your chances
of being wrong on this topic are substantially
greater than mine, no?

Just because you don't hear something


Never even climed one way or the other: YOU'RE
claiming the presence of physical effects that would
be trivailly easy to detect by objective means, effects
that have NEVER ONCE been detected in the
millions of measurements over many decades
and you provide not a single shred of evidence.
You make claims as if they were established
fact without a shred of evidence.

or know of no law of physics to support a
claim, does not make it untrue.


It's not up to me to support your extraordinary claim:
that's YOUR jobn, and one that you're not doing a
very good job of.

It just means that it hasn't been verified.


And that's YOUR job, sir, not mine.

Hop to it.

The world was once flat. Now it's round.


Boy, you picked the WRONG model.

The vast majority of the people in any know in
the world have known the world was not flat for
over 2000 years. Archimedes, in fact, calculated
the diameter of the earth to within a feqw percent
2 millenia ago. There was NO serious quesrtioning
of this information by anyone over that period of
time.

The ONLY ones who did NOT believe the world was
round where the uneducated and the religiously
superstitious, quite analogous to those who are not
well educated in electromagnetic theory who believe
in CD tweaks and the like, e.g., many high-end
audiophiles

Sorry, you raised the comparison, I only finished it.

The titanic was unsinkable.


Not according to the designers. They were able to
describe ahead of time several sceanrios where
the ship's integrity could be seriously compromised,
including ones not unnlike that that occurred. The
only ones who truly "believed" the ship was unsinkable
were those who didin't understand ship design and
physics, not unlike audiphiles who do not understand
electromagnetic theory that make wild claims about
how CDs can restore speakers.


Actually, the whole "unsinkable Titanic" myth can be traced to one remark
made by White Star Line managing director, J. Bruce Ismay following another,
rather more circumspect comment by the ship's captain, Edward John Smith. As
some prominent passengers were going up the gang-plank at Southampton,
someone asked Captain Smith if the new ship was was stronger than the last
generation of big liners (such as the 1906 Lusitannia), to which the Captain
replied "madam, Harlan and Wolf has designed this ship to be damn near
unsinkable" (notice the modifying phrase "damn near") To which Ismay
unfortunately added: "Why madam, the Lord Himself couldn't sink this ship."
Obvioulsy, "the Lord" was out to prove him wrong. :-

At any rate, no one else was ignorant enough to make such a claim, including
her designer, Harlan & Wolf naval architect Thomas Andrews.

  #50   Report Post  
Posted to rec.audio.high-end
Steven Sullivan Steven Sullivan is offline
external usenet poster
 
Posts: 1,268
Default Ultrabit Platium Disk Treatment

Codifus wrote:

Any skeptical taker who has the respect of Dick Pierce, Arny Kruger,
Sonnova, or Steve Sullivan. I chose you 4 because you have always been
so adamant about your beliefs based on facts. Nothing wrong with that at
all. It is a completely logical way to think. I'm just trying to open
some minds.


I'm flattered to be placed in such company, but I don't pretend to have a
fraction of the expertise in the physics of loudspeaker magnets that Mr.
Pierce has. I would suggest you pay close attention to the schooling he's
been giving you over the past few days in his replies.

My expertise is in the life sciences, and of necessity I'm well aware of
the need for and use of controls, when making truth-claims from experiment
and observation. The plain fact is that human mind is in some ways too
'open' to suggestion, and this is why we have need controls in the first
place. Carefully testing claims against reality is how science moves
forward. So far your experiments haven't begun to approach the standard
that would verify your claims, nor is here reason a priori, from previous
findings, to expect your claims to be true.




--
-S
A wise man, therefore, proportions his belief to the evidence. -- David Hume, "On Miracles"
(1748)



  #51   Report Post  
Posted to rec.audio.high-end
Sonnova Sonnova is offline
external usenet poster
 
Posts: 1,337
Default Ultrabit Platium Disk Treatment

On Sat, 26 Jul 2008 10:19:25 -0700, jwvm wrote
(in article ):

On Jul 25, 5:58*pm, Sonnova wrote:
Actually, they have absolutely nothing to do with each other. There is
nothing magnetic in a stamped, mass produced CD that would have the
slightest
affect on it. I was merely showing that one can magnetize and demagnetize
ferric objects using an AC signal , CDs aren't ferrous (except maybe
recordable ones which, if I understand correctly, do use a magnetic field to
effect the burning).


Its hard to follow what your problem is with Arnie's remarks.


Could be because I have no problems with Arnie's remarks.

Your
claim that it is possible to magnetize steel objects with an AC field
is true but irrelevant to the original claim regarding speakers given
their physical properties. The amount and polarity of the amount of
magnetism induced is quite random with an AC field. Indeed, it is
well-known that tape heads can be magnetized if one uses a
demagnetizer incorrectly.


Someone said that one couldn't magnetize objects with an AC field. I merely
said that such a claim wasn't necessarily so, and then provided an example.

Also, please note that CDRs and CDRWs do not employ magnetism when
recording. You are confusing them with magneto-optical media like
minidiscs:

http://en.wikipedia.org/wiki/Magneto-optical_drive


Wasn't too sure and stated same.

That would be more similar to anything you might achieve by playing a CD
through a stereo, except that the Weller iron operates with far higher
currents than a speaker voice coil.


They have nothing in common.


That is not true. The voice coil and the coil attached to a soldering
gun both generate an AC field. But as you seem to have indicated, the
AC field generated by a voice coil under normal operation will not
change the characteristics of a speaker in any significant manner.


Since I'm not sure of the exact context of the quote, above, I cannot
comment.

  #52   Report Post  
Posted to rec.audio.high-end
Sonnova Sonnova is offline
external usenet poster
 
Posts: 1,337
Default Ultrabit Platium Disk Treatment

On Sat, 26 Jul 2008 10:18:31 -0700, Arny Krueger wrote
(in article ):

"Codifus" wrote in message


You know, just as Arny Kruger mentioned deguassing old
real to reel tapes with a degausser and slowly moving
away,


I didn't say that, Sonnova did.


Yep.

this demagic CD is doing the exact same thing.


It can't because it lacks the ability to create an equally intense magnetic
field.


Not to mention that a stereo system is NOT a reel of magnetic tape.

  #53   Report Post  
Posted to rec.audio.high-end
Sonnova Sonnova is offline
external usenet poster
 
Posts: 1,337
Default Ultrabit Platium Disk Treatment

On Sat, 26 Jul 2008 10:17:59 -0700, Arny Krueger wrote
(in article ):

"Codifus" wrote in message


I'm just trying to open some minds.


I long ago learned that having an open mind is not facilitated by having
holes in the head. ;-)

Now, that taker should have a pair of
non-magnetically shielded speakers. I suggest these type
of speakers because they are easily set out of whack by
moving them around near a strong magnetic field like a
CRT based TV.


You can't hurt the speaker with the TV, but you may hurt the TV with the
speaker.

Dick Pierce just showed that with his comments on my comment about
magnetizing speakers via their pole pieces. If you can't magnetize or
demagnetize a speaker via a special fixture applied to its pole pieces, you
sure as the dickens aren't going to hurt it by putting it near a TV!

Usually the TV has the resources to fix itself in the form of a built-in
degaussing coil, often automatically engaged during power up, or via a
service function on some menu some place.


In the early color TV days, technicians used to carry large, hoola-hoop coils
of wire with them on their trucks. They would place the coils flat against
the screen and then, with them energized, pull them slowly away from the
screen. At some point, most would turn the coil perpendicular to the screen
and witch it off. At some point they built the coils into the TV and used a
shaped pulse with a slow release time (I surmise) to give the same effect as
backing the coil away from the screen. Haven't seen a screen degausser in
years, but I suspect that they are still around.
  #54   Report Post  
Posted to rec.audio.high-end
Sonnova Sonnova is offline
external usenet poster
 
Posts: 1,337
Default Ultrabit Platium Disk Treatment

On Sat, 26 Jul 2008 10:45:33 -0700, Steven Sullivan wrote
(in article ):

Codifus wrote:

Any skeptical taker who has the respect of Dick Pierce, Arny Kruger,
Sonnova, or Steve Sullivan. I chose you 4 because you have always been
so adamant about your beliefs based on facts. Nothing wrong with that at
all. It is a completely logical way to think. I'm just trying to open
some minds.


I'm flattered to be placed in such company, but I don't pretend to have a
fraction of the expertise in the physics of loudspeaker magnets that Mr.
Pierce has. I would suggest you pay close attention to the schooling he's
been giving you over the past few days in his replies.

My expertise is in the life sciences, and of necessity I'm well aware of
the need for and use of controls, when making truth-claims from experiment
and observation. The plain fact is that human mind is in some ways too
'open' to suggestion, and this is why we have need controls in the first
place. Carefully testing claims against reality is how science moves
forward. So far your experiments haven't begun to approach the standard
that would verify your claims, nor is here reason a priori, from previous
findings, to expect your claims to be true.


Well, said and I agree. Especially with what you said about paying attention
to Mr. Pierce's comments over the last few days. Very enlightening.
  #55   Report Post  
Posted to rec.audio.high-end
Codifus Codifus is offline
external usenet poster
 
Posts: 228
Default Ultrabit Platium Disk Treatment

On Jul 26, 1:17*pm, "Arny Krueger" wrote:
"Codifus" wrote in message



I'm just trying to open some minds.


I long ago learned that having an open mind is not facilitated by having
holes in the head. ;-)

Now, that taker should have a pair of
non-magnetically shielded speakers. I suggest these type
of speakers because they are easily set out of whack by
moving them around near a strong magnetic field like a
CRT based TV.


You can't hurt the speaker with the TV, but you may hurt the TV with the
speaker.

Dick Pierce just showed that with his comments on my comment about
magnetizing speakers via their pole pieces. If you can't magnetize or
demagnetize a speaker via a special fixture applied to its pole pieces, you
sure as the dickens aren't going to hurt it by putting it near a TV!

Usually the TV has the resources to fix itself in the form of a built-in
degaussing coil, often automatically engaged during power up, or via a
service function on some menu some place.


You're right. Forgive me. The TV has a miniscule magnetic field
relative to a speaker. I got it backwards. Still, I stand by my
observation that I heard the Demagic do its thing once. Most
definitely.

I had a pair of speakers which had shoddy imaging. To clarify that
test I placed them within 6 inches of each other, and verified that
they were correctly connected in phase. I listened to an audio source
in mono. The imaging was shot. With such a small distance between them
the ghost image midway between the speakers should have been very
strong.

It wasn't. So I played the Demagic CD and all was good again.

CD


  #56   Report Post  
Posted to rec.audio.high-end
Codifus Codifus is offline
external usenet poster
 
Posts: 228
Default Ultrabit Platium Disk Treatment

On Jul 26, 10:58*am, wrote:
On Jul 25, 10:56 pm, Codifus wrote:

Yes, I completely understand.


In fact, I believe the evidence you've presented shows
that you don't, with all respect.

When you move speakers past the TV, this
red and blue appears across the screen.
The effect of the speaker on the TV is much
more noticeable than any effect that may be
on the speaker. But there is an effect.


Again, you state this as if it was accepted fact.
It is not.


OK. I believe it so strongly and I'm prepared to prove it with the
tests you outlined.
Now, let's say that I guess evertyhing correctly, would my observaton
then become fact?


The CD is playing a tone. That tone manifests
itself as an electrical signal that goes through
your entire audio system. That electrical signal in
turn generates a magnetic field. The properties
of that generated magnetic field are such that it
neutralizes any static magnetism that may have
developed in the system.


Sorry, this is so much technobabble.


Like I said, I'm not expert in these fields. But I know you understand
what I'm trying to say.

Why and how is this signal in any way different
than EVERY OTHER SIGNAL that passes through
an audio system that integrates to 0?


First of all, some signals may suffer from a DC bias. Secondly, do you
always play music from beginning to end? Or could you at times, just
shut if off in the middle of a passage, or change channel etc?


Please explain how the VAST majority of materials
in an audio system "develop a static magnetism"
when they, in fact, do not posses a net polar magnetic
moment, materials such as copper, aluminum, plastic,
glass and such. I run a current by them of hundred,
thousands of amps, and they do NOT "develop a static
magnetism," how is it possible for current of millionths
of an amp to do it.


Here's my very non-technical, non expert opinion on the matter: it is
understood that all electronic components do not behave ideally. Far
from it. A resistor has some capacitance, capacitors have some
inductance etc. So I extrapolate that notion with the idea that any
component that can support a magnetic field would probably have a
tendency to store some magnetism. A copper wire, for example. The
amount it may store would be miniscule, but it may hold some. Same
goes for the capacitor, IC, etc. Now, all these components that are in
a speakers' crossover plus the voice coil may store just enough
magnetism to cause that phase issue I mentione before.

I may be wrong on the technical explanation, but I hope its enough to
convey the idea I'm trying to get across.

And how is it possible for those materials that COULD
"develop a static magnetism" (irons and steels) to
then have that reversed by currents of millionths of
an amp, far too small to overcome the coercivity in such
materials?

You know, just as Arny Kruger mentioned deguassing
old real to reel tapes with a degausser and slowly
moving away, this demagic CD is doing the exact same
thing.


No, it's not, and this is further evidence that you do
NOT understand it. It;s substantially different in at
least one important factor: the field generated by a
degausser if MILLIONS of time stronger than the field
generated by the currents inside your audio system.
It HAS to be to overcome the coercivity of the magnetic
material in the tape. If the impressed field DOES NOT
exceed a critical threshold by a wide margin, no change
in the magnetization of the material occurs. And a few
microamps of signal passing through an audio system,
even a few amps passing through a voice coil is FAR too
small to work.


Sorry, I said its the exact same thing. I should have been more clear
and said its the perfect analogy. When I post to a newsgroup, I just
post and rarely even double check for spelling etc. I'm not writing a
paper. It's mostly casual typing for me.

The magnitude of the filed is not what I'm comparing when looking at
the Demagic and tape degausser. It's the process. Both are generating
a magnetic field. In order for the deguassing process to work properly
one must move away very slowly. This is analogous to the Demagic CD
which fades away slowly. If you abruptly stop playing the CD in the
middle, it would be just like turning off the degausser suddenly
whilst in the middle of the degaussing process.


Think of the the tone as analogous to the
degausser's magnetic field. Now, as you
slowly move away from the tape
heads the Demagic tone slowly fades away.


That's all it is.


And you fell for such a simple, intuitive explanation
hook, line and sinker.

Too bad the explanation is simply wrong.


CD
  #57   Report Post  
Posted to rec.audio.high-end
Peter Wieck Peter Wieck is offline
external usenet poster
 
Posts: 2,418
Default Ultrabit Platium Disk Treatment

Mpfffffff... the first individual here to come up with a purely
geometrical process to trisect any angle - that person will have
credibility for me when it comes to little green pens, demagentizing
non-magnetic substances, bi-wiring speakers (assuming the original
single set of wires meets all electrical requirements based on
distance, gauge and material) - and so forth.

Otherwise, I have no need for additional smoke, mirrors or magic hats
in this household, thank you so much.

Peter Wieck
Melrose Park, PA
  #58   Report Post  
Posted to rec.audio.high-end
Sonnova Sonnova is offline
external usenet poster
 
Posts: 1,337
Default Ultrabit Platium Disk Treatment

On Sun, 27 Jul 2008 08:15:02 -0700, codifus wrote
(in article ):

On Jul 26, 10:58*am, wrote:
On Jul 25, 10:56 pm, Codifus wrote:

Yes, I completely understand.


In fact, I believe the evidence you've presented shows
that you don't, with all respect.

When you move speakers past the TV, this
red and blue appears across the screen.
The effect of the speaker on the TV is much
more noticeable than any effect that may be
on the speaker. But there is an effect.


Again, you state this as if it was accepted fact.
It is not.


OK. I believe it so strongly and I'm prepared to prove it with the
tests you outlined.
Now, let's say that I guess evertyhing correctly, would my observaton
then become fact?


The CD is playing a tone. That tone manifests
itself as an electrical signal that goes through
your entire audio system. That electrical signal in
turn generates a magnetic field. The properties
of that generated magnetic field are such that it
neutralizes any static magnetism that may have
developed in the system.


Sorry, this is so much technobabble.


Like I said, I'm not expert in these fields. But I know you understand
what I'm trying to say.

Why and how is this signal in any way different
than EVERY OTHER SIGNAL that passes through
an audio system that integrates to 0?


First of all, some signals may suffer from a DC bias.


Since most components aren't DC coupled, that would hardly matter. I'll
guarantee that no source (vinyl, CD player, tuner, tape deck,, etc.) has any
DC component in their signals even if both amp and preamp are DC coupled
throughout.

Secondly, do you
always play music from beginning to end? Or could you at times, just
shut if off in the middle of a passage, or change channel etc?


I don't see what relevance that has.


Please explain how the VAST majority of materials
in an audio system "develop a static magnetism"
when they, in fact, do not posses a net polar magnetic
moment, materials such as copper, aluminum, plastic,
glass and such. I run a current by them of hundred,
thousands of amps, and they do NOT "develop a static
magnetism," how is it possible for current of millionths
of an amp to do it.


Here's my very non-technical, non expert opinion on the matter: it is
understood that all electronic components do not behave ideally. Far
from it. A resistor has some capacitance, capacitors have some
inductance etc. So I extrapolate that notion with the idea that any
component that can support a magnetic field would probably have a
tendency to store some magnetism. A copper wire, for example. The
amount it may store would be miniscule, but it may hold some. Same
goes for the capacitor, IC, etc. Now, all these components that are in
a speakers' crossover plus the voice coil may store just enough
magnetism to cause that phase issue I mentione before.

I may be wrong on the technical explanation, but I hope its enough to
convey the idea I'm trying to get across.


Depends on the point that you're trying to get across. If your point is that
you have no idea what the "theory" behind this procedure is, and don't know
enough about electronics to understand that what you are proposing is simply
horse-puckey, then I'd say that you have conveyed your ideas very well.

And how is it possible for those materials that COULD
"develop a static magnetism" (irons and steels) to
then have that reversed by currents of millionths of
an amp, far too small to overcome the coercivity in such
materials?

You know, just as Arny Kruger mentioned deguassing
old real to reel tapes with a degausser and slowly
moving away, this demagic CD is doing the exact same
thing.


No, it's not, and this is further evidence that you do
NOT understand it. It;s substantially different in at
least one important factor: the field generated by a
degausser if MILLIONS of time stronger than the field
generated by the currents inside your audio system.
It HAS to be to overcome the coercivity of the magnetic
material in the tape. If the impressed field DOES NOT
exceed a critical threshold by a wide margin, no change
in the magnetization of the material occurs. And a few
microamps of signal passing through an audio system,
even a few amps passing through a voice coil is FAR too
small to work.


Sorry, I said its the exact same thing. I should have been more clear
and said its the perfect analogy. When I post to a newsgroup, I just
post and rarely even double check for spelling etc. I'm not writing a
paper. It's mostly casual typing for me.

The magnitude of the filed is not what I'm comparing when looking at
the Demagic and tape degausser. It's the process. Both are generating
a magnetic field. In order for the deguassing process to work properly
one must move away very slowly.


But the size of the field generated makes ALL the difference. Large amounts
of radium eventually killed Madame Curie, but many people wore luminous dial
watches on their wrists or in the watch pockets of their trousers for decades
with no ill effects. The difference is, of course, the amount of exposure to
the radium. Watches had miniscule amounts and these did not generally pose
much of a health hazard. OTOH, the workers at watch factories whose job it
was to apply the radium to the watch dials, had much higher incidents of
cancers than did the general population and of course, exposure to large
amounts of radium killed the aforementioned Marie Curie. Another example
would be for me to tell you to grab hold of the ends of two bare wires, the
other ends of which are soldered to a flashlight battery. Since the battery
is only 1.5 volts, you won't even feel it. Now I ask you to perform the exact
same procedure with bare ends of two wires that are plugged into a mains
outlet on the wall. Same procedure, different result.

This is analogous to the Demagic CD
which fades away slowly. If you abruptly stop playing the CD in the
middle, it would be just like turning off the degausser suddenly
whilst in the middle of the degaussing process.


Again you overlook the fact that the signal being generated by the CD is too
small to degauss anything - and that's assuming that there is anything there
to degauss in the first place.


Think of the the tone as analogous to the
degausser's magnetic field. Now, as you
slowly move away from the tape
heads the Demagic tone slowly fades away.


That's all it is.


And you fell for such a simple, intuitive explanation
hook, line and sinker.

Too bad the explanation is simply wrong.


CD


  #59   Report Post  
Posted to rec.audio.high-end
jwvm jwvm is offline
external usenet poster
 
Posts: 336
Default Ultrabit Platium Disk Treatment

On Jul 27, 11:15*am, codifus wrote:
Snip
Like I said, I'm not expert in these fields. But I know you understand
what I'm trying to say.


Its clear what you are trying to say but it does not make sense
technically. The actual properties of magnetism are very different
from what you describe.



First of all, some signals may suffer from a DC bias. Secondly, do you
always play music from beginning to end? Or could you at times, just
shut if off in the middle of a passage, or change channel etc?


Where would this bias come from? Normal stereo components do not have
response to DC so any DC component would be removed.


snip

Here's my very non-technical, non expert opinion on the matter: it is
understood that all electronic components do not behave ideally. Far
from it. A resistor has some capacitance, capacitors have some
inductance etc. So I extrapolate that notion with the idea that any
component that can support a magnetic field would probably have a
tendency to store some magnetism. A copper wire, for example. The
amount it may store would be miniscule, but it may hold some. Same
goes for the capacitor, IC, etc. Now, all these components that are in
a speakers' crossover plus the voice coil may store just enough
magnetism to cause that phase issue I mentione before.

*I may be wrong on the technical explanation, but I hope its enough to
convey the idea I'm trying to get across.

There is nothing wrong with your communication skills here. Its your
description of how the physical world works that is the problem.
Magnetism does not affect the vast majority of electronic components
in any significant fashion.


snip


The magnitude of the filed is not what I'm comparing when looking at
the Demagic and tape degausser. It's the process. Both are generating
a magnetic field. In order for the deguassing process to work properly
one must move away very slowly. This is analogous to the Demagic CD
which fades away slowly. If you abruptly stop playing the CD in the
middle, it would be just like turning off the degausser suddenly
whilst in the middle of the degaussing process.


the Demagic CD sounds like pure snake oil from your description as to
how it works. Its the perfect audiophile accessory. Given how it is
claimed to work, it would be difficult to perform a double-blind test
in a reasonable amount of time to prove that it really does nothing.


  #60   Report Post  
Posted to rec.audio.high-end
Blarp Blarp is offline
external usenet poster
 
Posts: 1
Default Ultrabit Platium Disk Treatment

Just another theory:

Somewhere in my humble abode I have a non hi-end sound system. (cd
player, stereo amp, modest speakers)

I find that re-seating the RCA connectors once in a while increases
sound quality (oxidized contacts sound somewhat like crossover
distortion).

The amp in casu developed a flaw in the speaker protection relay -
causing the same distortion as described above.

Adding a shot of sound at elevated volume cures it for some time
(higher current trough relay contacts ("contact wetting current")).

I suspect the playing any test-tone at elevated levels cures -in some
equipment- contact issues that slowly built up over time. (including
switches, pots etc.)

Also goes a long way to explain why "the new cable" sounds so much
better. The old one would too after just re-seating.

(incidentally - I observed gold flash finish on interconnects to be
very thin. I guess expensive interconnects / equipment should make a
difference specifically in this aspect..)



  #61   Report Post  
Posted to rec.audio.high-end
[email protected] dpierce.cartchunk.org@gmail.com is offline
external usenet poster
 
Posts: 334
Default Ultrabit Platium Disk Treatment

On Jul 27, 11:15 am, codifus wrote:
On Jul 26, 10:58 am, wrote:

On Jul 25, 10:56 pm, Codifus wrote:


Yes, I completely understand.


In fact, I believe the evidence you've presented shows
that you don't, with all respect.


When you move speakers past the TV, this
red and blue appears across the screen.
The effect of the speaker on the TV is much
more noticeable than any effect that may be
on the speaker. But there is an effect.


Again, you state this as if it was accepted fact.
It is not.


OK. I believe it so strongly and I'm prepared to
prove it with the tests you outlined.


Look, you have a problem comprehending what
I'm saying. Let me see if I can put it more clearly.

You, in fact, have TWO issues:

1. You've claimed you heard some audible effect
on using some contrivance,

2. You've made specific technical assertions about
the operation and behavior of the magnetic system
of loudspeakers, e.g.:

"I suggest these type of speakers because
they are easily set out of whack by moving them
around near a strong magnetic field like a CRT
based TV."

These two issue CAN be separated from one another,
and I have done that.

The first issue: what you may or may not have heard,
I am NOT addressing. The second issue: your
assertions about loudspeakers and electromagnetic
phenomenon, I AM addressing.

Please do not bundle the two together: they are
spearate issues.

Now, let's say that I guess evertyhing correctly,
would my observaton then become fact?


No, you just admitted to guessing. Why is a "guess"
equivalent to a "fact?"

The CD is playing a tone. That tone manifests
itself as an electrical signal that goes through
your entire audio system. That electrical signal in
turn generates a magnetic field. The properties
of that generated magnetic field are such that it
neutralizes any static magnetism that may have
developed in the system.


Sorry, this is so much technobabble.


Like I said, I'm not expert in these fields.


Then why try to act like one?

But I know you understand what I'm trying to say.


No, I don't know that. What I DO know is, with all
due respect, you do NOT know what you are talking
about when you make technical assertions about the
behavior of the electromagnetic properties of conductors,
magnets and loudspeakers.

Why and how is this signal in any way different
than EVERY OTHER SIGNAL that passes through
an audio system that integrates to 0?


First of all, some signals may suffer from a DC bias.


Give me a SINGLE example of ANY recorded signal
that exhibits such a property.

Secondly, do you always play music from beginning
to end? Or could you at times, just shut if off in the
middle of a passage, or change channel etc?


Not only incorrect, but irrelevant as well. You are making
the incorrect assumption that the entire system is
DC coupled from beginning to end. If it's not, all signals
integrate to 0.

Please explain how the VAST majority of materials
in an audio system "develop a static magnetism"
when they, in fact, do not posses a net polar magnetic
moment, materials such as copper, aluminum, plastic,
glass and such. I run a current by them of hundred,
thousands of amps, and they do NOT "develop a static
magnetism," how is it possible for current of millionths
of an amp to do it.


Here's my very non-technical, non expert opinion
on the matter:


And, as such, it is ONLY an opinion. If you are
under the impression that all technical opinions
are created equal, then you're mistaken.

it is understood that all electronic components
do not behave ideally.


No, amongst us practitioners, it is NOT so understood,
it is OBJECTIVELY VERIFIABLE.

That's one BIG difference between belief and science.

Far from it. A resistor has some capacitance, capacitors
have some inductance etc.


So? How is this relevant?

So I extrapolate that notion with the idea that any
component that can support a magnetic field
would probably have a tendency to store some
magnetism.


And, you're wrong. Extrapolation ONLY works
if the paradigm is linear and conitunous. You're
making the assumption that it is, when it, in fact,
is not.

A copper wire, for example. The amount it may
store would be miniscule, but it may hold some.


Then why has it not been found outside of the nutso
fringe of the audio "high-end?" There have been
measurement made on conductors and materials
at levels that are MANY orders of magntidue lower
than the lowest EVER encountered and the effect
you claim HAS NOT BEEN FOUND.

Your extrapolation is WRONG. Any conclusions made
on a wrong premise is likely to be wrong.

Same goes for the capacitor, IC, etc. Now, all these
components that are in a speakers' crossover plus
the voice coil may store just enough magnetism to
cause that phase issue I mentione before.


"stored magnetisom" in voice coils is trivially easy to
measure. I have the means right here at my disposal
to measure magnetic fields far lower than the fields
encountered inside speakers. Never has the effect
you claim been found.

I have the means of measuring phase to extraordinarily
small degrees, yet the "phase issue" you claim just is
not there.

And both of these measurements are at resolutions
FAR lower than are audible.

I may be wrong on the technical explanation,


Yes, unfortunately, you are.

but I hope its enough to convey the idea I'm
trying to get across.


They do: and they do so with sufficient clarity to
conclude with little doubt your idea is wrong.

Sorry, but that's the way it is.

The magnitude of the filed is not what I'm
comparing when looking at the Demagic
and tape degausser. It's the process.


You just don't get it. The magitude issue IS CRUCIAL
to the process. If the field does not exceed some critical
threshold, no magnetization takes effect. Below that, NO
effect occurs. And the level YOU'RE talking about are
MILLIONS of times too small.

Both are generating a magnetic field.


But fields that differ by ENORMOUS amounts, and that's
what you're refusing to get.

In order for the deguassing process to work properly
one must move away very slowly.


False. Wrong. Incorrect. How many more ways does on
have to say it before you begin to get it.

In order for the degaussing process to work. First, a field
WELL IN EXCESS and OPPOSITE polarity to the stored field
must be impressed. This gets the stored field to flip
polarity. And with each reverals, the field is diminished,
so that the polarity of the stored field keeps flipping, but the
magntidue diminshes.

But, once the impressed field drops below a certain point,
the process stops, and the material is LEFT with a small
residual field.

BUT THE MAGNITUDE OF THE FIELD STARTS OUT
MANY ORDERS OF MAGNITUDE HIGHER THAN
WHAT YOU'RE CLAIMING.

The field you're claiming CANNOT cause the degaussing,
because even at their stringest THEY ARE THOUSANDS
AND MILLIONS OF TIMES TOO SMALL.

Why do you refuse to understand that?

This is analogous to the Demagic CD
which fades away slowly. If you abruptly stop playing the CD in the
middle, it would be just like turning off the degausser suddenly
whilst in the middle of the degaussing process.


No, it is not, becaus ethe signals in your audio
system are simply FAR to small to have causes
the problem you claim in the first place, and far
too small to correct the problem which isn't there
to begin with.

And why don't the reverberation tails in recorded
music do EXACTLY the same thing? (Because,
just like the Demagic CD, they can't).

Look, you can firmly sit there an believe all you
want that you heard the effect. I'm not going to
argue with you nor challenge that assertion.

However, when you make specific TECHNICAL
assertions that are objectively verifiable about the
behavior of soudspeakers, and then tell us you're
no expert, and then continue to spout completely
unfounded technical assertion about how you
extrapolated a poor understanding in one domain
to completely incorrect assumptions in another,
then you're out of luck: those assertion will be
challenged.

This CD cannot possibly work via the mechanisms
you're claiming. Period.

By the way, did you EVER wonder why it's called
the De"MAGIC" CD?

  #63   Report Post  
Posted to rec.audio.high-end
Arny Krueger Arny Krueger is offline
external usenet poster
 
Posts: 17,262
Default Ultrabit Platium Disk Treatment

"codifus" wrote in message


OK. I believe it so strongly and I'm prepared to prove it
with the tests you outlined.


You're going to try some bias-controlled listening test?

Now, let's say that I guess evertyhing correctly, would
my observaton then become fact?


The phrase "Scientific fact" is an oxymoron. All findings of science are
provisional, and only relelvant until we find out something bettter.

The CD is playing a tone. That tone manifests
itself as an electrical signal that goes through
your entire audio system. That electrical signal in
turn generates a magnetic field. The properties
of that generated magnetic field are such that it
neutralizes any static magnetism that may have
developed in the system.


Whether you demagnetize it or not, an audio system is full of different
examples of what you call static magnetism.

For example, a CD player has at least two motors. The motors are usually
small DC motors with permanent magnet fields. These static magnetic fields
are a the most just a few inches from analog audio signal paths.

If static magnetism is such a sound quality problem, why hasn't every CD
player manufacturer ditched DC motors?

this is so much technobabble.


Agreed.

Like I said, I'm not expert in these fields. But I know
you understand what I'm trying to say.


It's like listening to someone trying to explain how their perpetual motion
machine works. They ain't gonna pass the next thermodynamics test, thinking
like that!

Why and how is this signal in any way different
than EVERY OTHER SIGNAL that passes through
an audio system that integrates to 0?


First of all, some signals may suffer from a DC bias.


First off, the audio production chain is full of 100.00% effective
roadblocks to DC bias.

For example, all performance spaces and studios have doors, windows, ducts
and very many accidental air leaks.

All microphones, even those with fully-enclosed chambers behind the
diaphragm, have carefully-designed air leaks.

All mic preamps, especially those that support phantom power, have
low-leakage input coupling capacitors.

All audio production gear has input, output, and interstage coupling
capacitors.

Most DAW software has commands for removing DC bias. It also has commands
that implement high pass filters that remove DC bias as part of their
inherent operation.

Recording hardware has input, output, and interstage coupling capacitors.

Ditto for the playback side.

Secondly, do you always play music from beginning to end?
Or could you at times, just shut if off in the middle of
a passage, or change channel etc?


If a recording has a DC bias, and you shut it off in the middle, there will
be a transient. That's one reason why audio production equipment, whether
hardware or software, is bound and determined to remove DC bias from the
recording.

Please explain how the VAST majority of materials
in an audio system "develop a static magnetism"
when they, in fact, do not posses a net polar magnetic
moment, materials such as copper, aluminum, plastic,
glass and such. I run a current by them of hundred,
thousands of amps, and they do NOT "develop a static
magnetism," how is it possible for current of millionths
of an amp to do it.


Good question.

Here's my very non-technical, non expert opinion on the
matter: it is understood that all electronic components
do not behave ideally. Far from it.


Wrong. Electronic components including the resistors and capactors that
tweeks obsess over, do in fact generally behave as ideally as is necessary
to provide sonically-transparent operation.

A resistor has some
capacitance, capacitors have some inductance etc.


The areas of non-ideal operation have to be large enough to be relevant.
Contrary to some people's beliefs, there really are thresholds of audibility
for non-ideal operation, and they are often relatively huge.

So I
extrapolate that notion with the idea that any component
that can support a magnetic field would probably have a
tendency to store some magnetism.


Problem is, there is a defined magnetic property of materials called
remanance, which describes that property.

http://en.wikipedia.org/wiki/Remanence

Many materials just don't have any remanance, and pure copper is one such
material.

Furthermore, remanance all by itself proves nothing. No inherent harm, no
inherent foul. There has to be some means by which the residual magnetic
field actually changes the audio signal to the degree that there is an
audible change.

As I pointed out above, audio gear can easily have very strong permanent
magnets built right into it. For example high end preamps with motorized
volume controls are very popular. What tweek has enhanced his high end
preamp by disabling the remote control faclity by removing the motor with
its *deadly* (!!!) permanant magnets. Do you realize how close the permanent
magnets in a motorized volume control are to the signal path? It doesn't
matter.

A copper wire, for example. The amount it may store would be miniscule,
but
it may hold some.


It is well known that every physical effect, every deadly poison, every
potential catastrophe has a threshold below which it is known to be
practically safe. You know that the granite countertops in your kitchen are
emitting potentially deadly Radon gas, right? Shall we get you a crowbar so
you can save the lives of you and your family and remove them immediately?

http://www.nytimes.com/2008/07/24/garden/24granite.html

Same goes for the capacitor, IC, etc.
Now, all these components that are in a speakers'
crossover plus the voice coil may store just enough
magnetism to cause that phase issue I mentione before.


It isn't significant. Speakers are so full of inherent phase shifts and time
delays that this is moot. Then we condescent to let the sound from the
speaker enter the listening room, and just about every allegedly bad time
delay and phase shift is swamped by room effects that are orders of
magnitude larger. Furthermore, the origional recording is full of similar
but even larger delays and phase shifts from the room in which the recording
was made.

I may be wrong on the technical explanation, but I hope
its enough to convey the idea I'm trying to get across.


What's coming accross is a basic priniciple of snake-oil audio - a complete
lack of understanding of the importance of quantifcation.

And how is it possible for those materials that COULD
"develop a static magnetism" (irons and steels) to
then have that reversed by currents of millionths of
an amp, far too small to overcome the coercivity in such
materials?


You know, just as Arny Kruger mentioned deguassing
old real to reel tapes with a degausser and slowly
moving away, this demagic CD is doing the exact same
thing.


That wasn't me, it was someone else. But he was talking about somthing
vastly different.

No, it's not, and this is further evidence that you do
NOT understand it. It;s substantially different in at
least one important factor: the field generated by a
degausser if MILLIONS of time stronger than the field
generated by the currents inside your audio system.
It HAS to be to overcome the coercivity of the magnetic
material in the tape. If the impressed field DOES NOT
exceed a critical threshold by a wide margin, no change
in the magnetization of the material occurs. And a few
microamps of signal passing through an audio system,
even a few amps passing through a voice coil is FAR too
small to work.


Agreed.

And, I know of no audiopiles who are ripping out all of their granite
countertops. Radon is deadly. It can cause lung cancer, as many miners have
discovered and died. Thing is, your kitchen isn't a Uranium mine.

Sorry, I said its the exact same thing. I should have
been more clear and said its the perfect analogy.


Taking it to be analogy completely misses the entirely valid issues that
were raised by Mr. Pierce.

If you want real fun, try to damage the data on floppy disc with a permanent
magnet. Modern magnetic recording materials have such high coercivity and
such excellent remanence of previouisly recorded information that it is
usualy mission impossbile.

The magnitude of the field is not what I'm comparing when
looking at the Demagic and tape degausser. It's the
process.


The processes aren't similar because the quantities are vastly different.

Both are generating a magnetic field.


All knives cut, but you must satisfy some rules of quantification and
actually draw the knife through something to cut it. Merely stroking the
knife through the air as if you were going to cut something does not
suffice.

In order for the deguassing process to work properly one must move
away very slowly.


For the degaussing process to work, there must be a magnetic field of
sufficient strength.

This is analogous to the Demagic CD
which fades away slowly. If you abruptly stop playing the
CD in the middle, it would be just like turning off the
degausser suddenly whilst in the middle of the degaussing
process.


If your alleged degaussing field is even a tiny bit too small, there is no
demagnetization. In this case the degaussing field is tens of times too
small.

Please get a 3.5" floppy and try to degauss it with an ordinary fridge
magnet. Please post again when you cause your first detectable data error.
Of course, make sure that the floppy had good data on it to begin with. ;-)



  #65   Report Post  
Posted to rec.audio.high-end
Sonnova Sonnova is offline
external usenet poster
 
Posts: 1,337
Default Ultrabit Platium Disk Treatment

On Mon, 28 Jul 2008 06:08:13 -0700, Arny Krueger wrote
(in article ):

"codifus" wrote in message


OK. I believe it so strongly and I'm prepared to prove it
with the tests you outlined.


You're going to try some bias-controlled listening test?

Now, let's say that I guess evertyhing correctly, would
my observaton then become fact?


The phrase "Scientific fact" is an oxymoron. All findings of science are
provisional, and only relelvant until we find out something bettter.

The CD is playing a tone. That tone manifests
itself as an electrical signal that goes through
your entire audio system. That electrical signal in
turn generates a magnetic field. The properties
of that generated magnetic field are such that it
neutralizes any static magnetism that may have
developed in the system.


Whether you demagnetize it or not, an audio system is full of different
examples of what you call static magnetism.

For example, a CD player has at least two motors. The motors are usually
small DC motors with permanent magnet fields. These static magnetic fields
are a the most just a few inches from analog audio signal paths.

If static magnetism is such a sound quality problem, why hasn't every CD
player manufacturer ditched DC motors?

this is so much technobabble.


Agreed.

Like I said, I'm not expert in these fields. But I know
you understand what I'm trying to say.


It's like listening to someone trying to explain how their perpetual motion
machine works. They ain't gonna pass the next thermodynamics test, thinking
like that!

Why and how is this signal in any way different
than EVERY OTHER SIGNAL that passes through
an audio system that integrates to 0?


First of all, some signals may suffer from a DC bias.


First off, the audio production chain is full of 100.00% effective
roadblocks to DC bias.

For example, all performance spaces and studios have doors, windows, ducts
and very many accidental air leaks.

All microphones, even those with fully-enclosed chambers behind the
diaphragm, have carefully-designed air leaks.

All mic preamps, especially those that support phantom power, have
low-leakage input coupling capacitors.

All audio production gear has input, output, and interstage coupling
capacitors.

Most DAW software has commands for removing DC bias. It also has commands
that implement high pass filters that remove DC bias as part of their
inherent operation.

Recording hardware has input, output, and interstage coupling capacitors.

Ditto for the playback side.

Secondly, do you always play music from beginning to end?
Or could you at times, just shut if off in the middle of
a passage, or change channel etc?


If a recording has a DC bias, and you shut it off in the middle, there will
be a transient. That's one reason why audio production equipment, whether
hardware or software, is bound and determined to remove DC bias from the
recording.

Please explain how the VAST majority of materials
in an audio system "develop a static magnetism"
when they, in fact, do not posses a net polar magnetic
moment, materials such as copper, aluminum, plastic,
glass and such. I run a current by them of hundred,
thousands of amps, and they do NOT "develop a static
magnetism," how is it possible for current of millionths
of an amp to do it.


Good question.

Here's my very non-technical, non expert opinion on the
matter: it is understood that all electronic components
do not behave ideally. Far from it.


Wrong. Electronic components including the resistors and capactors that
tweeks obsess over, do in fact generally behave as ideally as is necessary
to provide sonically-transparent operation.


Even if electronic components such as resistors and capacitors (and even
wire) did not behave ideally, where has anyone shown that residual magnetism
(called remanance) would (if present) result in some adverse sonic effect? It
would seem to me that if the folks at DeMagic have come up with a cure for
some audio ill, they would do well to at least show that the sonic ill they
striving to eliminate actually exists. Seems to me that they are making an
assumption that "residual magnetism" in components not only exists, but is
detrimental to a stereo system's sound and as far as I know that presumption
is not in evidence.


  #66   Report Post  
Posted to rec.audio.high-end
[email protected] outsor@city-net.com is offline
external usenet poster
 
Posts: 122
Default Ultrabit Platium Disk Treatment

"The flat earth analogy applies to extremists on both sides of the tired
debate, where faith and religion guide belief."

There is a debate but not an impasse.

One side has test results which show that as one toggles blind/sighted the
reported perception effects of the other sied toggle accordingly.

The other side offers instead "I hear it, I really do, don't you believe
me"? All the time the "hear" toggles as above.

There is a debate only in the sense that responses continue to be offered.
There is a confusion that a response, any response, substitutes for a
convincing response or one that can be shown to fit the facts at hand
independent of the one making a claim.

There is a difference of opinion not a real debate, and opinion can be had
a dime the dozen at any bar any night of the week.

There is an assertion of being on an equal footing as to what can be
demonstrated instead of from the perspective of "in my opinion" on the
part of the subjective radical.

We have had a gentleman's agreement of "can't we all just get along?", and
indeed we can as long as we accept what can be shown in a repeated fashion
independent of any individual claim can not be brushed away with "flat
earth" or "we are at an impasse" comments.

  #67   Report Post  
Posted to rec.audio.high-end
Sonnova Sonnova is offline
external usenet poster
 
Posts: 1,337
Default Ultrabit Platium Disk Treatment

On Mon, 28 Jul 2008 15:52:08 -0700, Arny Krueger wrote
(in article ):

"C. Leeds" wrote in message

wrote:

The ONLY ones who did NOT believe the world was
round where the uneducated and the religiously
superstitious, quite analogous to those who are not
well educated in electromagnetic theory who believe
in CD tweaks and the like, e.g., many high-end
audiophiles...


Yes, there are people who believe in demagnetizing completely non-magnetic
discs improves sound when there is no change in the data coming off the
disc, or its timing. A little chicken blood will make your stereo sound
better.


I've always found that a good Voodoo ceremony involving chicken blood and
chicken claws, especially around the speakers, is the way to go for getting
the most from them.

The flat earth analogy applies to extremists on both
sides of the tired debate, where faith and religion guide
belief.


So what is this religion that you keep talking about?


The religion of $4000 cables, disc demagnetizers, green pens to paint CDs,
Myrtlewood blocks placed on components, "treated" digital clocks plugged into
the mains supply, IEC mains cables the size of one's forearm, etc. It's
called the religion of the Lunatic Fringe Audiophile. Enid Lumley is its
High-Priestess and double-blind testing is it's anti-Christ.

We know what the anti-science extremists look like, what is the religion of
extreme science, and are there really any examples of it around here?


Science SHOULD BE the absence of religion although, sometimes well meaning
scientists can be led astray by exuberance too.

  #68   Report Post  
Posted to rec.audio.high-end
Codifus Codifus is offline
external usenet poster
 
Posts: 228
Default Ultrabit Platium Disk Treatment

On Jul 28, 9:08*am, "Arny Krueger" wrote:
"codifus" wrote in message



OK. I believe it so strongly and I'm prepared to prove it
with the tests you outlined.


You're going to try some bias-controlled listening test?


Im am totally willing. Dick Pierce outlined a perfect setup. Does it
meet with your approval?

Now, let's say that I guess evertyhing correctly, would
my observaton then become fact?


The phrase "Scientific fact" is an oxymoron. All findings of science are
provisional, and only relelvant until we find out something bettter.


OK, here' we go. I phrased that incorrectly. Let's say I get the
correct answer for everything, what would you all still have to be
skeptical about? I would like to get this done in a definitive fashion
that everyone agrees on.


The CD is playing a tone. That tone manifests
itself as an electrical signal that goes through
your entire audio system. That electrical signal in
turn generates a magnetic field. The properties
of that generated magnetic field are such that it
neutralizes any static magnetism that may have
developed in the system.


Whether you demagnetize it or not, an audio system is full of different
examples of what you call static magnetism.


For the sake of clarity, the only magnetism we are concerned with is
within the audio path, the most of which is in the speaker, crossover,
voice coil etc.


For example, a CD player has at least two motors. The motors are usually
small DC motors with permanent magnet fields. These static magnetic fields
are a the most just a few inches from analog audio signal paths.

If static magnetism is such a sound quality problem, why hasn't every CD
player manufacturer ditched DC motors?

this is so much technobabble.


Agreed.

Like I said, I'm not expert in these fields. But I know
you understand what I'm trying to say.


It's like listening to someone trying to explain how their perpetual motion
machine works. They ain't gonna pass the next thermodynamics test, thinking
like that!

Why and how is this signal in any way different
than EVERY OTHER SIGNAL that passes through
an audio system that integrates to 0?


I don't know how or why it happens, I just know that I have had
speakers that have lost their ability to image properly. Once
Demagicked, they recovered.

First of all, some signals may suffer from a DC bias.


First off, the audio production chain is full of 100.00% effective
roadblocks to DC bias.

For example, all performance spaces and studios have doors, windows, ducts
and very many accidental air leaks.

All microphones, even those with fully-enclosed chambers behind the
diaphragm, have carefully-designed air leaks.

All mic preamps, especially those that support phantom power, have
low-leakage input coupling capacitors.

All audio production gear has input, output, and interstage coupling
capacitors.

Most DAW software has commands for removing DC bias. It also has commands
that implement high pass filters that remove DC bias as part of their
inherent operation.

Recording hardware has input, output, and interstage coupling capacitors.

Ditto for the playback side.

Secondly, do you always play music from beginning to end?
Or could you at times, just shut if off in the middle of
a passage, or change channel etc?


If a recording has a DC bias, and you shut it off in the middle, there will
be a transient. That's one reason why audio production equipment, whether
hardware or software, is bound and determined to remove DC bias from the
recording.

Please explain how the VAST majority of materials
in an audio system "develop a static magnetism"
when they, in fact, do not posses a net polar magnetic
moment, materials such as copper, aluminum, plastic,
glass and such. I run a current by them of hundred,
thousands of amps, and they do NOT "develop a static
magnetism," how is it possible for current of millionths
of an amp to do it.


Good question.

Here's my very non-technical, non expert opinion on the
matter: it is understood that all electronic components
do not behave ideally. Far from it.


Wrong. Electronic components including the resistors and capactors that
tweeks obsess over, do in fact generally behave as ideally as is necessary
to provide sonically-transparent operation.


Actually, you are WRONG. How does something, in fact, GENERALLY behave
ideally? It either does or doesn't. So say it. Don't use this vague
statement to try to get by.

A resistor has some
capacitance, capacitors have some inductance etc.


The areas of non-ideal operation have to be large enough to be relevant.
Contrary to some people's beliefs, there really are thresholds of audibility
for non-ideal operation, and they are often relatively huge.


I don't agree or disagree.


So I
extrapolate that notion with the idea that any component
that can support a magnetic field would probably have a
tendency to store some magnetism.


Problem is, there is a defined magnetic property of materials called
remanance, which describes that property.

http://en.wikipedia.org/wiki/Remanence

Many materials just don't have any remanance, and pure copper is one such
material.


And all speakers are made of just copper, right?

Furthermore, remanance all by itself proves nothing. No inherent harm, no
inherent foul. There has to be some means by which the residual magnetic
field actually changes the audio signal to the degree that there is an
audible change.


I don't need proof, just the possibility. My observations show that
something is happening.

As I pointed out above, audio gear can easily have very strong permanent
magnets built right into it. For example high end preamps with motorized
volume controls are very popular. What tweek *has enhanced his high end
preamp by disabling the remote control faclity by removing the motor with
its *deadly* (!!!) permanant magnets. Do you realize how close the permanent
magnets in a motorized volume control are to the signal path? *It doesn't
matter.

OK, and?
A copper wire, for example. The amount it may store would be miniscule,
but
it may hold some.


It is well known that every physical effect, every deadly poison, every
potential catastrophe has a threshold below which it is known to be
practically safe. *You know that the granite countertops in your kitchen are
emitting potentially deadly Radon gas, right? Shall we get you a crowbar so
you can save the lives of you and your family and remove them immediately?


Don't doubt it. DId you also know that water can kill you if you drink
too much of it? I still drink alot of it. I'm a brave soul

http://www.nytimes.com/2008/07/24/garden/24granite.html

Same goes for the capacitor, IC, etc.
Now, all these components that are in a speakers'
crossover plus the voice coil may store just enough
magnetism to cause that phase issue I mentione before.


It isn't significant. Speakers are so full of inherent phase shifts and time
delays that *this is moot. Then we condescent to let the sound from the
speaker enter the listening room, and just about every allegedly bad time
delay and phase shift is swamped by room effects that are orders of
magnitude larger. Furthermore, the origional recording is full of similar
but even larger delays and phase shifts from the room in which the recording
was made.


All I am saying is that when a speaker has been suffuciently thrown
out of its specification such that it fails the mono test I keep
mentioning, playing the demagic CD sets it back. Music doesn't.


I may be wrong on the technical explanation, but I hope
its enough to convey the idea I'm trying to get across.


What's coming accross is a basic priniciple of snake-oil audio - a complete
lack of understanding of the importance of quantifcation.


I don't undertstand it, but I definitely heard it.

And how is it possible for those materials that COULD
"develop a static magnetism" (irons and steels) to
then have that reversed by currents of millionths of
an amp, far too small to overcome the coercivity in such
materials?


Like I said, you play the Demagic CD as loud as you comfrotably can.
The louder it is the more effective it is.


You know, just as Arny Kruger mentioned deguassing
old real to reel tapes with a degausser and slowly
moving away, this demagic CD is doing the exact same
thing.


That wasn't me, it was someone else. But he was talking about somthing
vastly different.

No, it's not, and this is further evidence that you do
NOT understand it. It;s substantially different in at
least one important factor: the field generated by a
degausser if MILLIONS of time stronger than the field
generated by the currents inside your audio system.
It HAS to be to overcome the coercivity of the magnetic
material in the tape. If the impressed field DOES NOT
exceed a critical threshold by a wide margin, no change
in the magnetization of the material occurs. And a few
microamps of signal passing through an audio system,
even a few amps passing through a voice coil is FAR too
small to work.


Give me a freaking break! A rocket ship and a cruise ship are both
what? Ships. Yet the speeds with which they travel are different by
orders of magnitude. Once warp drive is invented, you know what
they'll call the vessel that carries people across the galaxies at
those speeds? A starship. These examples go to show that an analogy
can apply even though one factor may different by an extremely large
amount. It's the same basic concept.


Agreed.

And, I know of no audiopiles who are ripping out all of their granite
countertops. Radon is deadly. It can cause lung cancer, as many miners have
discovered and died. Thing is, your kitchen isn't a Uranium mine.

Sorry, I said its the exact same thing. I should have
been more clear and said its the perfect analogy.


Taking it to be analogy completely misses the entirely valid issues that
were raised by Mr. Pierce.


I still stand by the analogy and am now much much more eager to prove
it.

If you want real fun, try to damage the data on floppy disc with a permanent
magnet. Modern magnetic recording materials have such high coercivity and
such excellent remanence of previouisly recorded information that it is
usualy mission impossbile.

The magnitude of the field is not what I'm comparing when
looking at the Demagic and tape degausser. It's the
process.


The processes aren't similar because the quantities are vastly different.

Both are generating a magnetic field.


All knives cut, but you must satisfy some rules of quantification and
actually draw the knife through something to cut it. Merely stroking the
knife through the air as if you were going to cut something does not
suffice.


What rule of quantification are referring to? And anyhow, as analogies
go, one thing is like another. It does not have to be exact. Rules are
only applied loosely.

In order for the deguassing process to work properly one must move
away very slowly.


For the degaussing process to work, there must be a magnetic field of
sufficient strength.

This is analogous to the Demagic CD
which fades away slowly. If you abruptly stop playing the
CD in the middle, it would be just like turning off the
degausser suddenly whilst in the middle of the degaussing
process.


If *your alleged degaussing field is even a tiny bit too small, there is no
demagnetization. *In this case the degaussing field is tens of times too
small.

Please get a *3.5" floppy and try to degauss it with an ordinary fridge
magnet. Please post again when you cause your first detectable data error.
Of course, make sure that the floppy had good data on it to begin with. ;-)


No, I will not. This is different. You're talking about it device that
was designed to hold data magnetically. A speaker is not designed to
hold data, period.

CD
  #69   Report Post  
Posted to rec.audio.high-end
Arny Krueger Arny Krueger is offline
external usenet poster
 
Posts: 17,262
Default Ultrabit Platium Disk Treatment

"Sonnova" wrote in message


Even if electronic components such as resistors and
capacitors (and even wire) did not behave ideally, where
has anyone shown that residual magnetism (called
remanance) would (if present) result in some adverse
sonic effect?


With sighted evaluations, all things are possible.

I first became aware of obsession with residual magnetism and magnetic
materials in the 1970s. At that time some Japanese high end manufacturers
were flogging people's ignorance in this manner.

It would seem to me that if the folks at
DeMagic have come up with a cure for some audio ill, they
would do well to at least show that the sonic ill they
striving to eliminate actually exists.


It appears that by letting the snow fall freely, they've been able to build
up a niche in their market.


Seems to me that
they are making an assumption that "residual magnetism"
in components not only exists, but is detrimental to a
stereo system's sound and as far as I know that
presumption is not in evidence.


Absolutely true. If anything, residual magnetism makes things work rather
than breaking things.


  #70   Report Post  
Posted to rec.audio.high-end
Harry Lavo Harry Lavo is offline
external usenet poster
 
Posts: 1,243
Default Ultrabit Platium Disk Treatment

wrote in message
...
"The flat earth analogy applies to extremists on both sides of the tired
debate, where faith and religion guide belief."

There is a debate but not an impasse.

One side has test results which show that as one toggles blind/sighted the
reported perception effects of the other sied toggle accordingly.

The other side offers instead "I hear it, I really do, don't you believe
me"? All the time the "hear" toggles as above.

There is a debate only in the sense that responses continue to be offered.
There is a confusion that a response, any response, substitutes for a
convincing response or one that can be shown to fit the facts at hand
independent of the one making a claim.

There is a difference of opinion not a real debate, and opinion can be had
a dime the dozen at any bar any night of the week.

There is an assertion of being on an equal footing as to what can be
demonstrated instead of from the perspective of "in my opinion" on the
part of the subjective radical.

We have had a gentleman's agreement of "can't we all just get along?", and
indeed we can as long as we accept what can be shown in a repeated fashion
independent of any individual claim can not be brushed away with "flat
earth" or "we are at an impasse" comments.


I am sorry, but it *is* an impasse.

The reason is, almost all claims of "science" with regard to audio
components revolve around ABX testing, or at least double-blind testing of
some sort. And while these tests are highly appropriate for audiometric
testing, they violate the cardinal principle of psychological test
design...they alter the variable under test...namely, listening for pleasure
and enjoyment (where differences and long term judgements and perceptions
arise from the sub-conscience). Good science, as opposed to handy science,
finds a way to design a test which measures the effect indirectly, in such
circumstances. At the very least, it designs and executes at least one such
test to validate that in fact the "shorthand" test does what it claims to do
in measuring the same thing with equally valid results. In the case of
audio, no organization has a financial interest in such testing, so it has
not been done. Its complexity and scope are beyond the logistical and
financial means of individuals.

When confronted with this issue, advocates of existing "shorthand"
methodology usually refuse to even acknowledge the issue, and usually end up
attacking the challengers as luddites. That is not science, it *is*
religion.

So we are at an impasse. The "scientists" say audio is a known art, and
our DBT tests prove it so. Others (and I am in this camp) say audio as it
applies to music reproduction may not be a completely known art; instead the
testing on its face may be at fault in promulgating this as myth. However,
such advocates lack the means of setting up the extensive and expensive test
required to get at, much less resolve, the issue.




  #71   Report Post  
Posted to rec.audio.high-end
bob bob is offline
external usenet poster
 
Posts: 670
Default Ultrabit Platium Disk Treatment

On Jul 29, 6:44*pm, "Harry Lavo" wrote:

I am sorry, but it *is* an impasse.


A "debate" in which one side has data, and the other side has no data,
is not properly termed an "impasse." It is properly termed a settled
question.

The reason is, almost all claims of "science" with regard to audio
components revolve around ABX testing, or at least double-blind testing of
some sort. *And while these tests are highly appropriate for audiometric
testing, they violate the cardinal principle of psychological test
design...they alter the variable under test...namely, listening for pleasure
and enjoyment (where differences and long term judgements and perceptions
arise from the sub-conscience).


If this were true, it would be a relevant consideration. But there is
absolutely no evidence that it is true. You cannot simply declare that
a test whose results you don't like has a flaw, Harry. You have to
demonstrate that flaw somehow. You never have.

That is why no one in the scientific community takes your objections
seriously.

bob

  #72   Report Post  
Posted to rec.audio.high-end
[email protected] khughes@nospam.net is offline
external usenet poster
 
Posts: 38
Default Ultrabit Platium Disk Treatment

Harry Lavo wrote:
wrote in message
...
"The flat earth analogy applies to extremists on both sides of the tired
debate, where faith and religion guide belief."

There is a debate but not an impasse.

One side has test results which show that as one toggles blind/sighted the
reported perception effects of the other sied toggle accordingly.

The other side offers instead "I hear it, I really do, don't you believe
me"? All the time the "hear" toggles as above.

There is a debate only in the sense that responses continue to be offered.
There is a confusion that a response, any response, substitutes for a
convincing response or one that can be shown to fit the facts at hand
independent of the one making a claim.

There is a difference of opinion not a real debate, and opinion can be had
a dime the dozen at any bar any night of the week.

There is an assertion of being on an equal footing as to what can be
demonstrated instead of from the perspective of "in my opinion" on the
part of the subjective radical.

We have had a gentleman's agreement of "can't we all just get along?", and
indeed we can as long as we accept what can be shown in a repeated fashion
independent of any individual claim can not be brushed away with "flat
earth" or "we are at an impasse" comments.


I am sorry, but it *is* an impasse.

The reason is, almost all claims of "science" with regard to audio
components revolve around ABX testing, or at least double-blind testing of
some sort.


Not to mention, of course, all the physics and engineering used to
*create* said audio components. Science/disciplines which typically
refute the possibility of the vast majority of disputed claims (e.g.
cable myths).

And while these tests are highly appropriate for audiometric
testing, they violate the cardinal principle of psychological test
design...they alter the variable under test...namely, listening for pleasure
and enjoyment (where differences and long term judgements and perceptions
arise from the sub-conscience).


This is a strawman you've been burning for years, and it is still a
strawman. You state that the variable under test is altered by the
methodology - you have no evidence of such. You can, as has been
pointed out to you ad nauseam, listen in any manner you want, for any
duration you want - months if you like - while doing an ABX study, or
any double blind A/B study.

I agree with your impasse assessment, however, since you clearly will
never accept this simple fact.

Good science, as opposed to handy science,
finds a way to design a test which measures the effect indirectly, in such
circumstances. At the very least, it designs and executes at least one such
test to validate that in fact the "shorthand" test does what it claims to do
in measuring the same thing with equally valid results. In the case of
audio, no organization has a financial interest in such testing, so it has
not been done. Its complexity and scope are beyond the logistical and
financial means of individuals.


Again you skate around the obvious. No such test validation is
required, when applied to the *specific* case. That is, the individual
who makes an extraordinary claim (lets say the efficacy of a mpingo disk
for example) based on a sighted evaluation, then uses the *EXACT* same
methodology adding only a blinding approach. Use A/B, ABChr, ABX,
relaxed casual listening, or ANY method you choose. The ONLY variable
is then the blinding. The whole concept of method validation is thus
moot, since the methods are identical, and the ONLY difference is lack
of visual clues. Absolutely no other variables are affected, and that
includes any performance anxiety related issues, insofar as the first
comparison - wherein the supposed difference was observed - was as much
a discrimination "test" as is the second, blinded comparison.

snip

Keith Hughes
  #73   Report Post  
Posted to rec.audio.high-end
bob bob is offline
external usenet poster
 
Posts: 670
Default Ultrabit Platium Disk Treatment

On Jul 29, 6:23*pm, codifus wrote:

OK, here' we go. I phrased that incorrectly. Let's say I get the
correct answer for everything, what would you all still have to be
skeptical about?


The cause of the effect. All we'd have is one experiment, the results
of which totally violate the laws of physics. Science wouldn't be
settled until we'd squared that circle. And the most likely
possibility would be that you'd done the experiment wrong, somehow.
Until you could explain *how* it works (and I do not mean hand-waving;
I mean demonstrating a measureable effect) there would still be plenty
to be skeptical about.

bob
  #74   Report Post  
Posted to rec.audio.high-end
Steven Sullivan Steven Sullivan is offline
external usenet poster
 
Posts: 1,268
Default Ultrabit Platium Disk Treatment

Harry Lavo wrote:
wrote in message
...
"The flat earth analogy applies to extremists on both sides of the tired
debate, where faith and religion guide belief."

There is a debate but not an impasse.

One side has test results which show that as one toggles blind/sighted the
reported perception effects of the other sied toggle accordingly.

The other side offers instead "I hear it, I really do, don't you believe
me"? All the time the "hear" toggles as above.

There is a debate only in the sense that responses continue to be offered.
There is a confusion that a response, any response, substitutes for a
convincing response or one that can be shown to fit the facts at hand
independent of the one making a claim.

There is a difference of opinion not a real debate, and opinion can be had
a dime the dozen at any bar any night of the week.

There is an assertion of being on an equal footing as to what can be
demonstrated instead of from the perspective of "in my opinion" on the
part of the subjective radical.

We have had a gentleman's agreement of "can't we all just get along?", and
indeed we can as long as we accept what can be shown in a repeated fashion
independent of any individual claim can not be brushed away with "flat
earth" or "we are at an impasse" comments.


I am sorry, but it *is* an impasse.


The reason is, almost all claims of "science" with regard to audio
components revolve around ABX testing, or at least double-blind testing of
some sort. And while these tests are highly appropriate for audiometric
testing, they violate the cardinal principle of psychological test
design...they alter the variable under test...namely, listening for pleasure
and enjoyment (where differences and long term judgements and perceptions
arise from the sub-conscience).


Nonsense, Harry. Go pick up a good book on experimental design. You'll
find that the idea of 'acclimation' is in there too, along with blind
testing. There is NOTHING to prevent your theoretical listener from
testing his imagination against reality...except fear of failure,
I suppose.


Good science, as opposed to handy science,
finds a way to design a test which measures the effect indirectly, in such
circumstances.


It might, but you have to make the good case that it's necessary.
And that the 'indirect' effect actually maps to the 'direct' effect.

But why would blind testing be any good for 'audiometry' if it fails
for audio comparison? And why would it be used in psychoacoustics
research at all?

The 'impasse', Harry,a rises from the endless reflex to
special pleading from a 'subjectivist' side that is often
fundamentally anti-science.


--
-S
A wise man, therefore, proportions his belief to the evidence. -- David Hume, "On Miracles"
(1748)

  #75   Report Post  
Posted to rec.audio.high-end
Harry Lavo Harry Lavo is offline
external usenet poster
 
Posts: 1,243
Default Ultrabit Platium Disk Treatment

"bob" wrote in message
...
On Jul 29, 6:44 pm, "Harry Lavo" wrote:

I am sorry, but it *is* an impasse.


A "debate" in which one side has data, and the other side has no data,
is not properly termed an "impasse." It is properly termed a settled
question.

The reason is, almost all claims of "science" with regard to audio
components revolve around ABX testing, or at least double-blind testing
of
some sort. And while these tests are highly appropriate for audiometric
testing, they violate the cardinal principle of psychological test
design...they alter the variable under test...namely, listening for
pleasure
and enjoyment (where differences and long term judgements and perceptions
arise from the sub-conscience).


If this were true, it would be a relevant consideration. But there is
absolutely no evidence that it is true. You cannot simply declare that
a test whose results you don't like has a flaw, Harry. You have to
demonstrate that flaw somehow. You never have.

That is why no one in the scientific community takes your objections
seriously.

bob


I rest my case. I guess true "scientists" don't consider the social
sciences as "science".

Bob, you don't do a test and then have to prove that it gets in the way. In
psychological and social science design work, you design the test from the
beginning so it *can't* get in the way, or at least appears most logically
that it *shouldn't* get in the way. ABX fails miserably in this regard. To
start with, the nature of the task is different (conscious discrimination as
opposed to unconscious detection). The listening conditions are different.
The musical context is usually different. The inability to train (because
you don't know in advance what you are listening for) is different. And I
could go on and on.

There is no substitute for numbers in this. The only way to design such a
test is to have the subject listen to music in as natural a setting as
possible, perhaps to monitor certain neurological stimulus while doing so,
certainly to have scalar monadic rating after the fact, and then to comparee
LARGE NUMBERS of respondents across carefully matched samples. So thiere is
no CONSCIOUS discrimination involved. This has never been done to validate
individual double-blind discriminatory testing, and until it is done, the
test is simply an unproven vehicle for purposes of detecting musical
differences.





  #76   Report Post  
Posted to rec.audio.high-end
[email protected] outsor@city-net.com is offline
external usenet poster
 
Posts: 122
Default Ultrabit Platium Disk Treatment

We have had a gentleman's agreement of "can't we all just get along?",
and
indeed we can as long as we accept what can be shown in a repeated

fashion
independent of any individual claim can not be brushed away with "flat
earth" or "we are at an impasse" comments.


I am sorry, but it *is* an impasse.

The reason is, almost all claims of "science" with regard to audio
components revolve around ABX testing, or at least double-blind testing of
some sort. And while these tests are highly appropriate for audiometric
testing, they violate the cardinal principle of psychological test
design...they alter the variable under test...namely, listening for
pleasure and enjoyment (where differences and long term judgements and
perceptions arise from the sub-conscience). Good science, as opposed to
handy science, finds a way to design a test which measures the effect
indirectly, in such circumstances. At the very least, it designs and
executes at least one such test to validate that in fact the "shorthand"
test does what it claims to do in measuring the same thing with equally
valid results."

I am sorry, this is but hand waving and special pleading. It reminds one
of the instant fall back defense when esp claimants routinely fail to
demonstrate by controlled testing their "powers". They say something to
the effect that the bad vibes from the skeptics present for the test
destroys the special context in which the powers are normally present.

If any thing such as the above claim is to be given equal consideration, it
is the burden of the claimant to demonstrate it.

That audio testing by listening alone works is conceded. To move the goal
posts by speculating that some other factor "X" is not being evaluated has
yet to be shown. No such is done where other human perception events such
as with the other senses are being evaluated. There is no reason to think
hearing is an exception.

Please read to the end for a discussion of the claimed "impasse".

"In the case of audio, no organization has a financial interest in such
testing, so it has not been done. Its complexity and scope are beyond the
logistical and financial means of individuals.

When confronted with this issue, advocates of existing "shorthand"
methodology usually refuse to even acknowledge the issue, and usually end
up attacking the challengers as luddites. That is not science, it *is*
religion.

So we are at an impasse. The "scientists" say audio is a known art, and
our DBT tests prove it so. Others (and I am in this camp) say audio as it
applies to music reproduction may not be a completely known art; instead
the testing on its face may be at fault in promulgating this as myth.
However, such advocates lack the means of setting up the extensive and
expensive test required to get at, much less resolve, the issue."

Leaving aside the red herring and strawman notions and barbs above, let us
turn to the claim of an impasse still existing.


I think not. It is like two teams of cancer researchers using different
methods and meeting at 6 months to evaluate results. Because results are
similar they declare an impasse as to which method is best in long term. At
12 months they meet again and one method is fading with a clear method
continuing to work. The impasse is declared broken.

At that moment a new person rushes in to declare that he speculates that
given a new method involving an as unyet shown factor "x" that has occurred
to him results will be different. He declares that there exists once again
the same impasse as 6 months ago until his new speculation is tested. But
it is unlikely to occur because of demands of time and money leaving the
impasse in place.

I think not. There is a body of test results with routinely similar
results showing that reported subjective perception events which toggle on
and off as the test is blind and sighted. That is the current state of
things and no other results on an equal footing nor at an impasse are
there to be considered.

  #77   Report Post  
Posted to rec.audio.high-end
Harry Lavo Harry Lavo is offline
external usenet poster
 
Posts: 1,243
Default Ultrabit Platium Disk Treatment

wrote in message
...

snip


I think not. There is a body of test results with routinely similar
results showing that reported subjective perception events which toggle on
and off as the test is blind and sighted. That is the current state of
things and no other results on an equal footing nor at an impasse are
there to be considered.


But nobody has corraborated that the test itself is not the reason the
sighted differences do not hold. That is the crux of the matter....until
the test (which is interventionist in nature) can be proven to provide
identical results to much more expensive and sophisticated testing that is
not interventionist, then the test itself has to be consider potentially
suspect. THAT is just good science.


  #78   Report Post  
Posted to rec.audio.high-end
[email protected] khughes@nospam.net is offline
external usenet poster
 
Posts: 38
Default Ultrabit Platium Disk Treatment

Harry Lavo wrote:
"bob" wrote in message
...
On Jul 29, 6:44 pm, "Harry Lavo" wrote:
I am sorry, but it *is* an impasse.

A "debate" in which one side has data, and the other side has no data,
is not properly termed an "impasse." It is properly termed a settled
question.

The reason is, almost all claims of "science" with regard to audio
components revolve around ABX testing, or at least double-blind testing
of
some sort. And while these tests are highly appropriate for audiometric
testing, they violate the cardinal principle of psychological test
design...they alter the variable under test...namely, listening for
pleasure
and enjoyment (where differences and long term judgements and perceptions
arise from the sub-conscience).

If this were true, it would be a relevant consideration. But there is
absolutely no evidence that it is true. You cannot simply declare that
a test whose results you don't like has a flaw, Harry. You have to
demonstrate that flaw somehow. You never have.

That is why no one in the scientific community takes your objections
seriously.

bob


I rest my case. I guess true "scientists" don't consider the social
sciences as "science".


Yes, they do. That's why they design discrimination tests for
discrimination, and they don't use *preference/acceptability* testing to
determine whether a *difference* exists.


Bob, you don't do a test and then have to prove that it gets in the way. In
psychological and social science design work, you design the test from the
beginning so it *can't* get in the way, or at least appears most logically
that it *shouldn't* get in the way. ABX fails miserably in this regard.


In your opinion. A micro-minority opinion based on available data.

To
start with, the nature of the task is different (conscious discrimination as
opposed to unconscious detection).


A false constraint *you* apply. Not one inherent in the test. Sit back
and enjoy "A" for as long as you like. Sit back and enjoy "B" for as
long as you like. Sit back and enjoy "X" for as long as you like. At
the end, rate each using whatever scale you want. Do a significant
number of trials, and "X" should be easily identifiable as either "A" or
"B". A statistically significant level of correlation with the
correct A/B is a positive. No correlation, or correlation with the
incorrect A/B indicates a negative result. Exactly as your "monadic"
testing would do.

The listening conditions are different.


A false constraint *you* apply. Not one inherent in the test.

The musical context is usually different.


A false constraint *you* apply. Not one inherent in the test.

The inability to train (because
you don't know in advance what you are listening for) is different.


A totally bogus argument. In virtually ALL cases where this particular
debate arises, the subjective "difference" has already been observed,
under non-controlled conditions. So, clearly, *you* (as in the claimant
of the difference) are fully aware of what you are listening for.
You've already identified its character when present, and the character
when its lacking.

And I could go on and on.


And you have, you have.

There is no substitute for numbers in this.


This is just nonsense. The "numbers" are in; the vast majority of
people do not hear significant differences in cables, cable elevators,
rocks, etc. These products are not "niche" products because they matter
to large numbers of people.

Typically, here, we're starting with the *claim* by some individual or
group that some tweek has produced a subjectively identifiable physical
result. A result for which there is no known scientific basis, and
often just the opposite. One need only test this individual/group using
an appropriate methodology to prove/disprove (with a margin of error)
the accuracy of the claim. There is NO need to try and extrapolate this
to the general population, so test population size is irrelevant.

The only way to design such a
test is to have the subject listen to music in as natural a setting as
possible, perhaps to monitor certain neurological stimulus while doing so,
certainly to have scalar monadic rating after the fact, and then to comparee
LARGE NUMBERS of respondents across carefully matched samples. So thiere is
no CONSCIOUS discrimination involved.


Well, ignoring that test methodologies used for *preference* and for
*acceptability* are ill suited in the extreme for the purpose at hand,
please name even ONE instance where monadic testing does not require
conscious discrimination. For your claim to be true, participants would
have to be unaware that they were participating in a test (else they are
constantly comparing current product/stimuli to past experiences of
different product/stimuli), they have to have no *interest* in
determining whether the current product/stimuli is *acceptable* in
performance (unlikely in the extreme), and of course, they *cannot be
questioned about their impressions after the test concludes*. Such
questioning forces the CONSCIOUS discrimination (either acceptability
against personal criteria, or comparison to previous experience) you
continually rail against.

This has never been done to validate
individual double-blind discriminatory testing, and until it is done, the
test is simply an unproven vehicle for purposes of detecting musical
differences.


OSAF. All scientific studies on subjective data, that I'm aware of, are
conducted blind unless brand recognition is part of the design. This
includes all drug and medical device testing, even when there are
numerous objective criteria that can be, and are measured concurrent
with the subjective evaluations. That blind tests don't confirm your
results in no way invalidates them.

Keith Hughes

  #79   Report Post  
Posted to rec.audio.high-end
Arny Krueger Arny Krueger is offline
external usenet poster
 
Posts: 17,262
Default Ultrabit Platium Disk Treatment

"Harry Lavo" wrote in message


Bob, you don't do a test and then have to prove that it
gets in the way.


More to the point, its not very wise to use a test that is known to be
subject to excessive false standard as your universal and final reference.

In psychological and social science
design work, you design the test from the beginning so it
*can't* get in the way, or at least appears most
logically that it *shouldn't* get in the way.


ABX was developed for exactly that reason. We tried a few DBT protocols, and
they got in the way. We then implemented ABX and found that it does a great
job of getting out of the way. ABX gets out of the way better than sighted
evaluations.

ABX fails miserably in this regard.


By what standard?

To start with, the nature of
the task is different (conscious discrimination as
opposed to unconscious detection).


Where is it written that listening pleasure as determined by *conscious*
perceptions is invalid or incomplete?

Is there any absolute proof that there even is such a thing as the
unconscious mind.

Is there a means for determining the state of this purported unconscious
mind in a reliable manner?

The listening conditions are different.


ABX does not change the listening conditions.

The musical context is usually different.


ABX does not change the musical context.

The inability to train (because you don't
know in advance what you are listening for) is different.


The essence of ABX is the availability of a known training reference. That
is what alternatives A and B are all about.

And I could go on and on.


But would there be anything but one man's speculation?



  #80   Report Post  
Posted to rec.audio.high-end
Sonnova Sonnova is offline
external usenet poster
 
Posts: 1,337
Default Ultrabit Platium Disk Treatment

On Tue, 29 Jul 2008 15:23:11 -0700, codifus wrote
(in article ):

On Jul 28, 9:08*am, "Arny Krueger" wrote:
"codifus" wrote in message



OK. I believe it so strongly and I'm prepared to prove it
with the tests you outlined.


You're going to try some bias-controlled listening test?


Im am totally willing. Dick Pierce outlined a perfect setup. Does it
meet with your approval?

Now, let's say that I guess evertyhing correctly, would
my observaton then become fact?


The phrase "Scientific fact" is an oxymoron. All findings of science are
provisional, and only relelvant until we find out something bettter.


OK, here' we go. I phrased that incorrectly. Let's say I get the
correct answer for everything, what would you all still have to be
skeptical about? I would like to get this done in a definitive fashion
that everyone agrees on.


The CD is playing a tone. That tone manifests
itself as an electrical signal that goes through
your entire audio system. That electrical signal in
turn generates a magnetic field. The properties
of that generated magnetic field are such that it
neutralizes any static magnetism that may have
developed in the system.


Whether you demagnetize it or not, an audio system is full of different
examples of what you call static magnetism.


For the sake of clarity, the only magnetism we are concerned with is
within the audio path, the most of which is in the speaker, crossover,
voice coil etc.


For example, a CD player has at least two motors. The motors are usually
small DC motors with permanent magnet fields. These static magnetic fields
are a the most just a few inches from analog audio signal paths.

If static magnetism is such a sound quality problem, why hasn't every CD
player manufacturer ditched DC motors?

this is so much technobabble.


Agreed.

Like I said, I'm not expert in these fields. But I know
you understand what I'm trying to say.


It's like listening to someone trying to explain how their perpetual motion
machine works. They ain't gonna pass the next thermodynamics test, thinking
like that!

Why and how is this signal in any way different
than EVERY OTHER SIGNAL that passes through
an audio system that integrates to 0?


I don't know how or why it happens, I just know that I have had
speakers that have lost their ability to image properly. Once
Demagicked, they recovered.

First of all, some signals may suffer from a DC bias.


First off, the audio production chain is full of 100.00% effective
roadblocks to DC bias.

For example, all performance spaces and studios have doors, windows, ducts
and very many accidental air leaks.

All microphones, even those with fully-enclosed chambers behind the
diaphragm, have carefully-designed air leaks.

All mic preamps, especially those that support phantom power, have
low-leakage input coupling capacitors.

All audio production gear has input, output, and interstage coupling
capacitors.

Most DAW software has commands for removing DC bias. It also has commands
that implement high pass filters that remove DC bias as part of their
inherent operation.

Recording hardware has input, output, and interstage coupling capacitors.

Ditto for the playback side.

Secondly, do you always play music from beginning to end?
Or could you at times, just shut if off in the middle of
a passage, or change channel etc?


If a recording has a DC bias, and you shut it off in the middle, there will
be a transient. That's one reason why audio production equipment, whether
hardware or software, is bound and determined to remove DC bias from the
recording.

Please explain how the VAST majority of materials
in an audio system "develop a static magnetism"
when they, in fact, do not posses a net polar magnetic
moment, materials such as copper, aluminum, plastic,
glass and such. I run a current by them of hundred,
thousands of amps, and they do NOT "develop a static
magnetism," how is it possible for current of millionths
of an amp to do it.


Good question.

Here's my very non-technical, non expert opinion on the
matter: it is understood that all electronic components
do not behave ideally. Far from it.


Wrong. Electronic components including the resistors and capactors that
tweeks obsess over, do in fact generally behave as ideally as is necessary
to provide sonically-transparent operation.


Actually, you are WRONG. How does something, in fact, GENERALLY behave
ideally? It either does or doesn't. So say it. Don't use this vague
statement to try to get by.


There is nothing inconsistent here. An example of something generally
behaving ideally would be a component that behaves in a perfectly linear and
non-compromising manner in the passband where it is being used, but outside
of that passband may display non-linearity, too much capacitance, or too much
inductance, etc.

A resistor has some
capacitance, capacitors have some inductance etc.


The areas of non-ideal operation have to be large enough to be relevant.
Contrary to some people's beliefs, there really are thresholds of audibility
for non-ideal operation, and they are often relatively huge.


I don't agree or disagree.


So I
extrapolate that notion with the idea that any component
that can support a magnetic field would probably have a
tendency to store some magnetism.


Problem is, there is a defined magnetic property of materials called
remanance, which describes that property.

http://en.wikipedia.org/wiki/Remanence

Many materials just don't have any remanance, and pure copper is one such
material.


And all speakers are made of just copper, right?


actually, MOST speakers are made of paper.

Furthermore, remanance all by itself proves nothing. No inherent harm, no
inherent foul. There has to be some means by which the residual magnetic
field actually changes the audio signal to the degree that there is an
audible change.


I don't need proof, just the possibility. My observations show that
something is happening.


Your "sighted" observations.

As I pointed out above, audio gear can easily have very strong permanent
magnets built right into it. For example high end preamps with motorized
volume controls are very popular. What tweek *has enhanced his high end
preamp by disabling the remote control faclity by removing the motor with
its *deadly* (!!!) permanant magnets. Do you realize how close the permanent
magnets in a motorized volume control are to the signal path? *It doesn't
matter.

OK, and?
A copper wire, for example. The amount it may store would be miniscule,
but
it may hold some.


It is well known that every physical effect, every deadly poison, every
potential catastrophe has a threshold below which it is known to be
practically safe. *You know that the granite countertops in your kitchen are
emitting potentially deadly Radon gas, right? Shall we get you a crowbar so
you can save the lives of you and your family and remove them immediately?


Don't doubt it. DId you also know that water can kill you if you drink
too much of it? I still drink alot of it. I'm a brave soul

http://www.nytimes.com/2008/07/24/garden/24granite.html

Same goes for the capacitor, IC, etc.
Now, all these components that are in a speakers'
crossover plus the voice coil may store just enough
magnetism to cause that phase issue I mentione before.


It isn't significant. Speakers are so full of inherent phase shifts and time
delays that *this is moot. Then we condescent to let the sound from the
speaker enter the listening room, and just about every allegedly bad time
delay and phase shift is swamped by room effects that are orders of
magnitude larger. Furthermore, the origional recording is full of similar
but even larger delays and phase shifts from the room in which the recording
was made.


All I am saying is that when a speaker has been suffuciently thrown
out of its specification such that it fails the mono test I keep
mentioning, playing the demagic CD sets it back. Music doesn't.


But you fail to explain what could possibly throw a speaker that far out of
kilter. This side of frying it with too much drive, I can't think of a thing.


I may be wrong on the technical explanation, but I hope
its enough to convey the idea I'm trying to get across.


What's coming accross is a basic priniciple of snake-oil audio - a complete
lack of understanding of the importance of quantifcation.


I don't undertstand it, but I definitely heard it.


You BELIEVE you heard it.

And how is it possible for those materials that COULD
"develop a static magnetism" (irons and steels) to
then have that reversed by currents of millionths of
an amp, far too small to overcome the coercivity in such
materials?


Like I said, you play the Demagic CD as loud as you comfrotably can.
The louder it is the more effective it is.


You know, just as Arny Kruger mentioned deguassing
old real to reel tapes with a degausser and slowly
moving away, this demagic CD is doing the exact same
thing.


That wasn't me, it was someone else. But he was talking about somthing
vastly different.

No, it's not, and this is further evidence that you do
NOT understand it. It;s substantially different in at
least one important factor: the field generated by a
degausser if MILLIONS of time stronger than the field
generated by the currents inside your audio system.
It HAS to be to overcome the coercivity of the magnetic
material in the tape. If the impressed field DOES NOT
exceed a critical threshold by a wide margin, no change
in the magnetization of the material occurs. And a few
microamps of signal passing through an audio system,
even a few amps passing through a voice coil is FAR too
small to work.


Give me a freaking break! A rocket ship and a cruise ship are both
what? Ships. Yet the speeds with which they travel are different by
orders of magnitude. Once warp drive is invented, you know what
they'll call the vessel that carries people across the galaxies at
those speeds? A starship. These examples go to show that an analogy
can apply even though one factor may different by an extremely large
amount. It's the same basic concept.


This is irrelevant to the point. Quantitative things have a threshold below
which they are ineffective. Try to cook a roast in an oven set to 120
degrees. It won't cook, even though 120 degrees of internal temperature is
the ideal temperature of medium-rare beef. You have to raise the temperature
to over 212 degrees before the beef will cook. Under that and the meat will
spoil, not cook.

A huge magnetic field might ruin a loudspeaker, but a tiny one, below the
threshold of permeability is like cooking a roast at 120 degrees. Nothing
will happen.


Agreed.

And, I know of no audiopiles who are ripping out all of their granite
countertops. Radon is deadly. It can cause lung cancer, as many miners have
discovered and died. Thing is, your kitchen isn't a Uranium mine.

Sorry, I said its the exact same thing. I should have
been more clear and said its the perfect analogy.


Taking it to be analogy completely misses the entirely valid issues that
were raised by Mr. Pierce.


I still stand by the analogy and am now much much more eager to prove
it.

If you want real fun, try to damage the data on floppy disc with a permanent
magnet. Modern magnetic recording materials have such high coercivity and
such excellent remanence of previouisly recorded information that it is
usualy mission impossbile.

The magnitude of the field is not what I'm comparing when
looking at the Demagic and tape degausser. It's the
process.


The processes aren't similar because the quantities are vastly different.

Both are generating a magnetic field.


All knives cut, but you must satisfy some rules of quantification and
actually draw the knife through something to cut it. Merely stroking the
knife through the air as if you were going to cut something does not
suffice.


What rule of quantification are referring to? And anyhow, as analogies
go, one thing is like another. It does not have to be exact. Rules are
only applied loosely.

In order for the deguassing process to work properly one must move
away very slowly.


For the degaussing process to work, there must be a magnetic field of
sufficient strength.

This is analogous to the Demagic CD
which fades away slowly. If you abruptly stop playing the
CD in the middle, it would be just like turning off the
degausser suddenly whilst in the middle of the degaussing
process.


If *your alleged degaussing field is even a tiny bit too small, there is no
demagnetization. *In this case the degaussing field is tens of times too
small.

Please get a *3.5" floppy and try to degauss it with an ordinary fridge
magnet. Please post again when you cause your first detectable data error.
Of course, make sure that the floppy had good data on it to begin with. ;-)


No, I will not. This is different. You're talking about it device that
was designed to hold data magnetically. A speaker is not designed to
hold data, period.

CD


He's trying to show you the difference in the sizes of various magnetic
fields and how they determine the extent to which those things that are in
proximity to those fields are affected - something you seem bound and
determined not to acknowledge.

Reply
Thread Tools
Display Modes

Posting Rules

Smilies are On
[IMG] code is On
HTML code is Off


Similar Threads
Thread Thread Starter Forum Replies Last Post
Recording level low - Nomrd Jukdbox Zen Extra to Audigy 2ZS Platium Pro # Fred # Pro Audio 10 October 2nd 06 06:27 PM
Iso Booth Treatment [email protected] Pro Audio 3 December 8th 05 07:20 PM
Which treatment for that guitar ? Bontempi Pro Audio 9 September 14th 04 05:20 AM
Wall treatment ScottW Audio Opinions 9 December 18th 03 10:01 PM
Wall Treatment ScottW High End Audio 0 December 16th 03 07:26 PM


All times are GMT +1. The time now is 12:42 AM.

Powered by: vBulletin
Copyright ©2000 - 2024, Jelsoft Enterprises Ltd.
Copyright ©2004-2024 AudioBanter.com.
The comments are property of their posters.
 

About Us

"It's about Audio and hi-fi"